Sie sind auf Seite 1von 23

1. GR NO. 82027 (ROMARICO G. VITUG v.

CA) Issue: Whether or not the amount from the survivorship But although the survivorship agreement is per
G.R. No. 82027 March 29, 1990 agreement was part of the petitioner Vitug and the se not contrary to law its operation or effect may
ROMARICO G. VITUG, petitioner, decedent’s conjugal property? be violative of the law. For instance, if it be
vs. shown in a given case that such agreement is a
THE HONORABLE COURT OF APPEALS and Held: NO. The Court ruled that a Survivorship mere cloak to hide an inofficious donation, to
ROWENA FAUSTINO-CORONA, respondents. Agreement is neither a donation mortis causa nor a transfer property in fraud of creditors, or to
donation inter vivos, because it was to take effect after defeat the legitime of a forced heir, it may be
Previous Case: Involved the probate of the two wills of the death of one party. Secondly, it is not a donation assailed and annulled upon such grounds.
the late Dolores Luchangco Vitug, who died in New York, between the spouses because it involved no conveyance
U. S.A., and named private respondent Rowena of a spouse's own properties to the other. There is no demonstration here that the survivorship
Faustino-Corona as executrix. In that previous decision, agreement had been executed for such unlawful
it upheld the appointment of Nenita Alonte as co-special It is in the nature of an aleatory contract whereby one or purposes, or, as held by the respondent court, in order to
administrator of Mrs. Vitug's estate with her (Mrs. Vitug's) both of the parties reciprocally bind themselves to give or frustrate our laws on wills, donations, and conjugal
widower, petitioner Romarico G. Vitug, pending probate. to do something in consideration of what the other shall partnership.
give or do upon the happening of an event which is to
Facts: Romarico G. Vitug filed a motion asking for occur at an indeterminate time or is uncertain, such as 2. G.R. No. 167330 (PHILIPPINE HEALTH CARE
authority from the probate court to sell certain shares of death. Under Article 2010 of the Code: PROVIDERS, INC v. CIR)
stock and real properties belonging to the estate to cover
allegedly his advances to the estate in the sum of ART. 2010. By an aleatory contract, FACTS:
P667,731.66, alleging that such amount was his one of the parties or both reciprocally bind
personal funds, allegedly obtained from a survivorship themselves to give or to do something in
agreement executed between him, his late wife, and the On January 27, 2000, respondent Commissioner of
consideration of what the other shall give or do Internal Revenue sent petitioner a formal demand letter
Bank of American National Trust Savings upon the happening of an event which is
Association.The said agreement contained the following and the corresponding assessment notices demanding
uncertain, or which is to occur at an the payment of deficiency taxes, including surcharges
stipulations: indeterminate time. and interest, for the taxable years 1996 and 1997 in the
total amount of P224,702,641.18.
(1) All money deposited and to be deposited Under the aforequoted provision, the fulfillment of an
with the Bank in their joint savings current aleatory contract depends on either the happening of an
account shall be both their property and shall be The deficiency DST (documentary stamp tax)
event which is (1) "uncertain," (2) "which is to occur at an assessment was imposed on petitioner's health care
payable to and collectible or withdrawable by indeterminate time."
either or any of them during their lifetime; and agreement with the members of its health care program
pursuant to Section 185 of the 1997 Tax Code.
A survivorship agreement, the sale of a sweepstake
(2) After the death of one of them, the same ticket, a transaction stipulating on the value of currency, Section 185. Stamp tax on fidelity bonds and other
shall belong to and be the sole property of the and insurance have been held to fall under the first insurance policies. - On all policies of insurance or
surviving spouse and payable to and collectible category, while a contract for life annuity or pension bonds or obligations of the nature of indemnity for
or withdrawable by such survivor under Article 2021, et sequentia, has been categorized loss, damage, or liability made or renewed by any
under the second. In either case, the element of risk is person, association or company or corporation
Rowena Corona opposed the motion to sell on the present. In the case at bar, the risk was the death of one transacting the business of accident, fidelity,
employer's liability, plate, glass, steam boiler, burglar,
ground that the same funds withdrawn from savings party and survivorship of the other.
elevator, automatic sprinkler, or other branch of
account were conjugal partnership properties and part of insurance (except life, marine, inland, and fire
the estate, and hence, there was allegedly no ground for The conclusion is accordingly unavoidable that Mrs. insurance), and all bonds, undertakings, or
reimbursement. She also sought his ouster for failure to Vitug having predeceased her husband, the latter has recognizances, conditioned for the performance of the
include the sums in question for inventory and for acquired upon her death a vested right over the amounts duties of any office or position, for the doing or not
"concealment of funds belonging to the estate." doing of anything therein specified, and on all
under savings account No. 35342-038 of the Bank of
obligations guaranteeing the validity or legality of any
America. Insofar as the respondent court ordered their bond or other obligations issued by any province, city,
The trial court upheld the survivorship agreement and inclusion in the inventory of assets left by Mrs. Vitug, we municipality, or other public body or organization, and
granted the motion to sell. hold that the court was in error. Being the separate on all obligations guaranteeing the title to any real
property of petitioner, it forms no more part of the estate, or guaranteeing any mercantile credits, which
estate of the deceased. may be made or renewed by any such person,
The Court of Appeals, in the petition for certiorari filed by
company or corporation, there shall be collected a
the herein private respondent, held that the above- documentary stamp tax of fifty centavos (P0.50) on
quoted survivorship agreement constitutes a conveyance Additional Notes: each four pesos (P4.00), or fractional part thereof, of
mortis causa which "did not comply with the formalities of the premium charged. (emphasis supplied)
a valid will as prescribed by Article 805 of the Civil Code” However, the Court has warned:
Petitioner protested the assessment in a letter dated offered at exchanges for the transaction of the business.
February 23, 2000. As respondent did not act on the Petitioner's health care agreement is primarily a It is an excise on the facilities used in the transaction of
protest, petitioner filed a petition for review in the Court contract of indemnity. And in the recent case of Blue the business, separate and apart from the business
of Tax Appeals (CTA) seeking the cancellation of the Cross Healthcare, Inc. v. Olivares, this Court ruled that a itself.
deficiency VAT and DST assessments. health care agreement is in the nature of a non-life
insurance policy. What is Documentary Stamp Tax?
On April 5, 2002, the CTA rendered a decision partially
granting the petition for review. Respondent appealed Contrary to petitioner's claim, its health care agreement The DST is levied on the exercise by persons of certain
the CTA decision to the CA insofar as it cancelled the is not a contract for the provision of medical services. privileges conferred by law for the creation, revision, or
DST assessment. He claimed that petitioner's health Petitioner does not actually provide medical or hospital termination of specific legal relationships through the
care agreement was a contract of insurance subject to services but merely arranges for the same and pays for execution of specific instruments. It is an excise upon the
DST under Section 185 of the 1997 Tax Code. them up to the stipulated maximum amount of coverage. privilege, opportunity, or facility offered at exchanges for
It is also incorrect to say that the health care agreement the transaction of the business.
On August 16, 2004, the CA rendered its decision. It held is not based on loss or damage because, under the said 3. GR No. 67835 (MALAYAN INSURANCE CO. v.
that petitioner's health care agreement was in the nature agreement, petitioner assumes the liability and GREGORIA CRUZ ARNALDO)
of a non-life insurance contract subject to DST. Petitioner indemnifies its member for hospital, medical and related
moved for reconsideration but the CA denied it. Hence, expenses (such as professional fees of physicians). The No. L-67835. October 12, 1987.*
this petition. term "loss or damage" is broad enough to cover the MALAYAN INSURANCE CO., INC. (MICO), petitioner,
monetary expense or liability a member will incur in case us.GREGORIA CRUZ ARNALDO, in her capacity as
Petitioner essentially argues that its health care of illness or injury. the INSURANCE COMMISSIONER, and
agreement is not a contract of insurance but a contract CORONACION PINCA, respondents.
for the provision on a prepaid basis of medical services, Under the health care agreement, the rendition of
including medical check-up, that are not based on loss or hospital, medical and professional services to the
damage. Petitioner also insists that it is not engaged in member in case of sickness, injury or emergency or his Facts: On June 7, 1981, the petitioner Malayan
the insurance business. It is a health maintenance availment of so-called "out-patient services" (including Insurance Co.(MICO) issued to the private respondent,
organization regulated by the Department of Health, not physical examination, x-ray and laboratory tests, medical Coronacion Pinca, Fire Insurance Policy No. F-001-
an insurance company under the jurisdiction of the consultations, vaccine administration and family planning 17212 on her property for the amount of P100,000.00,
Insurance Commission. For these reasons, petitioner counseling) is the contingent event which gives rise to effective July 22, 1981, until July 22, 1982.
asserts that the health care agreement is not subject to liability on the part of the member. In case of exposure of
DST. the member to liability, he would be entitled to On October 15, 1981, MICO allegedly cancelled the
indemnification by petitioner. policy for non-payment, of the premium and sent the
ISSUE: corresponding notice to Pinca.
Furthermore, the fact that petitioner must relieve its
Is a health care agreement in the nature of an insurance member from liability by paying for expenses arising from On December 24, 1981, payment of the premium for
contract and therefore subject to the documentary stamp the stipulated contingencies belies its claim that its Pinca was received by Domingo Adora, agent of MICO.
tax (DST) imposed under Section 185 of Republic Act services are prepaid. The expenses to be incurred by
8424 (Tax Code of 1997)? each member cannot be predicted beforehand, if they On January 15, 1982, Adora remitted this payment to
can be predicted at all. Petitioner assumes the risk of MICO, together with other payments.
paying for the costs of the services even if they are On January 18, 1982, Pinca's property was completely
HELD: significantly and substantially more than what the burned.
member has "prepaid." Petitioner does not bear the costs
Yes. In particular, the DST under Section 185 of the alone but distributes or spreads them out among a large On February 5, 1982, Pinca's payment was returned by
1997 Tax Code is imposed on the privilege of making or group of persons bearing a similar risk, that is, among all MICO to Adora on the ground that her policy had been
renewing any policy of insurance (except life, marine, the other members of the health care program. This is cancelled earlier. But Adora refused to accept it.
inland and fire insurance), bond or obligation in the insurance.
nature of indemnity for loss, damage, or liability. In due time, Pinca made the requisite demands for
Petitioner's contention that it is a health maintenance payment, which MICO rejected. She then went to the
Under the law, a contract of insurance is an organization and not an insurance company is irrelevant. Insurance Commission. It is because she was ultimately
agreement whereby one undertakes for a Contracts between companies like petitioner and the sustained by the public respondent that the petitioner
consideration to indemnify another against loss, beneficiaries under their plans are treated as insurance has come to us for relief.
damage or liability arising from an unknown or contracts.
contingent event. The event insured against must be
designated in the contract and must either be unknown Moreover, DST is not a tax on the business transacted Issue: Whether Pinca is entitled to claim the insurance
or contingent. but an excise on the privilege, opportunity, or facility premiums
having failed in his effort, he filed the action for the
Ruling: Yes. The SC held that there was an existing And it is a well-known principle under the law of agency recovery of the same before the Court of First Instance
insurance at the time of the loss was sustained by Pinca. that: of Cebu, which rendered the adverse decision as earlier
Payment was made, rendering the policy operative as of "Payment to an agent having authority to receive or referred to against both petitioners.
June 22, 1981, and removing it from the provisions of collect payment is equivalent to payment to the principal
Article 77. Thereafter, the policy could be cancelled on himself; such payment is complete when the money ISSUE: Whether or not the respondent is entitled to the
any of the supervening grounds enumerated in Article 64 delivered is into the agent's hands and is a discharge of insurance
(except "nonpayment of premium") provided the the indebtedness owing to the principal."
cancellation was made in accordance therewith and with HELD: No. Since petitioner Pacific Life disapproved the
Article 65. 4. G.R. No. L-31845 April 30, 1979 insurance application of respondent Ngo Hing, the
GREAT PACIFIC LIFE ASSURANCE COMPANY vs. binding deposit receipt in question had never become in
The petitioner relies heavily on Section 77 of the HONORABLE COURT OF APPEALS force at any time.
Insurance Code providing that:
"SEC. 77, An insurer is entitled to payment of the FACTS: It appears that on March 14, 1957, private As held by this Court, where an agreement is made
premium as soon as the thing is exposed to the peril respondent Ngo Hing filed an application with the Great between the applicant and the agent, no liability shall
insured against. Notwithstanding any agreement to the Pacific Life Assurance Company (hereinafter referred to attach until the principal approves the risk and a receipt
contrary, no policy or contract of insurance issued by an as Pacific Life) for a twenty-year endowment policy in the is given by the agent. The acceptance is merely
insurance company is valid and binding unless and until amount of P50,000.00 on the life of his one-year old conditional and is subordinated to the act of the company
the premium thereof has been paid, except in the case of daughter Helen Go. Said respondent supplied the in approving or rejecting the application. Thus, in life
a life or an industrial life policy whenever the grace essential data which petitioner Lapulapu D. Mondragon, insurance, a "binding slip" or "binding receipt" does not
period provision applies." Branch Manager of the Pacific Life in Cebu City wrote on insure by itself
the corresponding form in his own handwriting.
The above provision is not applicable because payment Mondragon finally type-wrote the data on the application
of the premium was in fact eventually made in this case. form which was signed by private respondent Ngo Hing. It bears repeating that through the intra-company
Notably, the premium invoice issued to Pinca at the time The latter paid the annual premium the sum of P1,077.75 communication of April 30, 1957 Pacific Life disapproved
of the delivery of the policy on June 7, 1981 was going over to the Company, but he retained the amount the insurance application in question on the ground that
stamped "Payment Received" of the amount of P930.60 of P1,317.00 as his commission for being a duly it is not offering the twenty-year endowment insurance
on "12-24-81" by Domingo Adora.14 This is important authorized agent of Pacific Life. Upon the payment of policy to children less than seven years of age. What it
because it suggests an understanding between MICO the insurance premium, the binding deposit receipt was offered instead is another plan known as the Juvenile
and the insured that such payment could be made later, issued to private respondent Ngo Hing. Likewise, Triple Action, which private respondent failed to accept.
as agent Adora had assured Pinca. In any event, it is not petitioner Mondragon handwrote at the bottom of the In the absence of a meeting of the minds between
denied that this payment was actually made by Pinca to back page of the application form his strong petitioner Pacific Life and private respondent Ngo Hing
Adora, who remitted the same to MICO. The payment recommendation for the approval of the insurance over the 20-year endowment life insurance in the amount
was made on December 24, 1981, and the fire occurred application. Then on April 30, 1957, Mondragon received of P50,000.00 in favor of the latter's one-year old
on January 18, 1982. a letter from Pacific Life disapproving the insurance daughter, and with the non-compliance of the above
application. The letter stated that the said life insurance quoted conditions stated in the disputed binding deposit
application for 20-year endowment plan is not available receipt, there could have been no insurance contract
It is not disputed that the premium was actually paid by duly perfected between then Acordingly, the deposit paid
Pinca to Adora on December 24, 1981, who received it for minors below seven years old, but Pacific Life can
consider the same under the Juvenile Triple Action Plan, by private respondent shall have to be refunded by
on behalf of MICO, to which it was remitted on January Pacific Life.
15, 1982. What is questioned is the validity of Pinca's and advised that if the offer is acceptable, the Juvenile
payment and of Adora's authority to receive it. Non-Medical Declaration be sent to the company.
MICO's acknowledgment of Adora as its agent defeats The Supreme Court held that it is not impressed with
its contention that he was not authorized to receive the The non-acceptance of the insurance plan by Pacific Life private respondent's contention that failure of
premium payment on its behalf. It is clearly provided in was allegedly not communicated by petitioner petitioner Mondragon to communicate to him the
Section 306 of the Insurance Code that: Mondragon to private respondent Ngo Hing. Instead, on rejection of the insurance application would not have
May 6, 1957, Mondragon wrote back Pacific Life again any adverse effect on the allegedly perfected
strongly recommending the approval of the 20-year temporary contract In this first place, there was no
endowment insurance plan to children, pointing out that contract perfected between the parties who had no
"SEC. 306. x x x x x x x x x meeting of their minds. Private respondent, being an
"Any insurance company which delivers to an insurance since 1954 the customers, especially the Chinese, were
asking for such coverage. authorized insurance agent of Pacific Life at Cebu
agent or insurance broker a policy or contract of branch office, is indubitably aware that said
insurance shall be deemed to have authorized such company does not offer the life insurance applied
agent or broker to receive on its behalf payment of any It was when things were in such state that on May 28, for. When he filed the insurance application in dispute,
premium which is due on such policy or contract of 1957 Helen Go died of influenza with complication of private respondent was, therefore, only taking the
insurance at the time of its issuance or delivery or which bronchopneumonia. Thereupon, private respondent chance that Pacific Life will approve the recommendation
becomes due thereon." sought the payment of the proceeds of the insurance, but
of Mondragon for the acceptance and approval of the Fieldmen's Insurance Company (FIC) agent Benjamin … . lt would now rely on the fact that the insured owned
application in question along with his proposal that the Sambat to apply for a Common Carrier's Liability a private vehicle, not a common carrier, something which
insurance company starts to offer the 20-year Insurance Policy covering his motor vehicle. As testified it knew all along when not once but twice its agent, no
endowment insurance plan for children less than seven by Songco’s son Amor later, he said that their vehicle is doubt without any objection in its part, exerted the utmost
years. Nonetheless, the record discloses that Pacific Life an ‘owner’ private vehicle and not for passengers, but pressure on the insured, a man of scant education, to
had rejected the proposal and recommendation. agent Sambat said that they can insure whatever kind of enter into such a contract.
Secondly, having an insurable interest on the life of his vehicle because their company is not owned by the
one-year old daughter, aside from being an insurance government, so they could do what they please
agent and an offense associate of petitioner Mondragon, whenever they believe a vehicle is insurable. FIC Inc.
2. Fieldmen’s Insurance incurred legal liability under
private respondent Ngo Hing must have known and issued a Common Carriers Accident Insurance Policy
the policy.
followed the progress on the processing of such with a duration of 1 year. Upon expiration, FIC Inc, the
application and could not pretend ignorance of the policy was renewed. During the effectivity of the renewed
Company's rejection of the 20-year endowment life policy, the insured vehicle while being driven by Rodolfo Since some of the conditions contained in the policy
insurance application. Songco, a duly licensed driver and son of Federico (the issued by the defendant-appellant were impossible to
vehicle owner) collided with a car where Federico and comply with under the existing conditions at the time and
Rodolfo died, while Carlos Songco and his wife Angelita, 'inconsistent with the known facts,' the insurer 'is
This Court is of the firm belief that private respondent
and a family friend sustained physical injuries. estopped from asserting breach of such conditions.’
had deliberately concealed the state of health and
Except for the fact, that they were not fare paying
physical condition of his daughter Helen Go. Where
passengers, their status as beneficiaries under the policy
private respondent supplied the required essential Issue:
is recognized therein.
data for the insurance application form, he was fully
aware that his one-year old daughter is typically a 1. Whether Fieldmen’s Insurance Company (FIC)
mongoloid child. Such a congenital physical defect Even if it be assumed that there was an ambiguity,
is estopped from enforcing forfeitures in its
could never be ensconced nor distinguished. ambiguities or obscurities must be strictly interpreted
favor- YES
Nonetheless, private respondent, in apparent bad against the party that caused them ( Qua Chee Gan v.
2. Whether FIC can escape liability under a
faith, withheld the fact material to the risk to be Law Union and Rock Insurance Co., Ltd.)
common carrier insurance policy on the pretext
assumed by the insurance company. As an that what was insured was a private vehicle and
insurance agent of Pacific Life, he ought to know, as not a common carrier, the policy being issued The contract of insurance is one of perfect good faith
he surely must have known. his duty and upon the agent’s insistence.- NO (uberima fides) not for the insured alone, but equally so
responsibility to such a material fact. Had he diamond for the insurer; in fact, it is more so for the latter, since its
said significant fact in the insurance application form dominant bargaining position carries with it stricter
Pacific Life would have verified the same and would Ruling:
responsibility."
have had no choice but to disapprove the application
outright. 1. FIC is "estopped from enforcing forfeitures in its
6.
favor, in order to forestall fraud or imposition on
G.R. No. 154514. July 28, 2005
Whether intentional or unintentional the concealment the insured.
WHITE GOLD MARINE SERVICES, INC., Petitioners,
entitles the insurer to rescind the contract of insurance. vs. PIONEER INSURANCE AND SURETY
Private respondent appears guilty thereof. In Qua Chee Gan v. Law Union and Rock Insurance CORPORATION AND THE STEAMSHIP MUTUAL
Co., Ltd., It was held that where inequitable conduct is UNDERWRITING ASSOCIATION (BERMUDA) LTD.,
SC was constrained to hold that no insurance contract shown by an insurance firm, it is "estopped from Respondents.
was perfected between the parties with the enforcing forfeitures in its favor, in order to forestall fraud
noncompliance of the conditions provided in the binding or imposition on the insured.”
FACTS: White Gold Marine Services, Inc.
receipt, and concealment, as legally defined, having procured a protection and indemnity coverage for its
been combated by herein private respondent. After petitioner Fieldmen's Insurance Co., Inc. had led vessels from The Steamship Mutual Underwriting
5. G.R. No. L-24833 (FIELDMEN'S INSURANCE CO., the insured Federico Songco to believe that he could Association (Bermuda) Limited (Steamship Mutual)
INC. v. MERCEDES VARGAS VDA. DE SONGCO, ET qualify under the common carrier liability insurance through Pioneer Insurance and Surety Corporation.
AL. and CA) policy, and to enter into contract of insurance paying the Subsequently, White Gold was issued a Certificate of
premiums due, it could not, thereafter, in any litigation Entry and Acceptance. Pioneer also issued receipts
FIELDMEN'S INSURANCE CO., INC., vs. MERCEDES arising out of such representation, be permitted to evidencing payments for the coverage. When White Gold
VARGAS VDA. DE SONGCO, ET AL. and COURT OF change its stand to the detriment of the heirs of the failed to fully pay its accounts, Steamship Mutual refused
APPEALS insured. As estoppel is primarily based on the doctrine of to renew the coverage.
G.R. No. L-24833 September 23, 1968 good faith and the avoidance of harm that will befall the
FERNANDO, J.: innocent party due to its injurious reliance, the failure to
Steamship Mutual thereafter filed a case against White
apply it in this case would result in a gross travesty of
Gold for collection of sum of money to recover the latter’s
Facts: Federico Songco, a man of scant education [first justice
unpaid balance. White Gold on the other hand, filed a
grader], owned a private jeepney. He was induced by
complaint before the Insurance Commission claiming because the issue in Hyopsung was the jurisdiction of
that Steamship Mutual violated Sections 186 and 187 of the court over Hyopsung. Relatedly, a mutual insurance company is a cooperative
the Insurance Code, while Pioneer violated Sections enterprise where the members are both the insurer and
299, 300 and 301 in relation to Sections 302 and 303, ISSUES: (1) Is Steamship Mutual, a P & I Club, insured. In it, the members all contribute, by a system of
thereof. engaged in the insurance business in the Philippines? - premiums or assessments, to the creation of a fund from
YES which all losses and liabilities are paid, and where the
The Insurance Commission dismissed the complaint. It (2) Does Pioneer need a license as an profits are divided among themselves, in proportion to
said that there was no need for Steamship Mutual to insurance agent/broker for Steamship Mutual? - their interest. Additionally, mutual insurance
secure a license because it was not engaged in the YES associations, or clubs, provide three types of coverage,
insurance business. It explained that Steamship Mutual namely, protection and indemnity, war risks, and defense
was a Protection and Indemnity Club (P & I Club). RULING: costs.
Likewise, Pioneer need not obtain another license as
insurance agent and/or a broker for Steamship Mutual A P & I Club is "a form of insurance against third party
because Steamship Mutual was not engaged in the 1.) Section 2(2) of the Insurance Code enumerates
what constitutes "doing an insurance business" or liability, where the third party is anyone other than the P
insurance business. Moreover, Pioneer was already & I Club and the members." By definition then,
licensed, hence, a separate license solely as "transacting an insurance business". These are:
Steamship Mutual as a P & I Club is a mutual insurance
agent/broker of Steamship Mutual was already association engaged in the marine insurance business.
superfluous. (a) making or proposing to make, as insurer, any
insurance contract;
(b) making, or proposing to make, as surety, any The records reveal Steamship Mutual is doing business
The Court of Appeals affirmed the decision of the contract of suretyship as a vocation and not as merely in the country albeit without the requisite certificate of
Insurance Commissioner. In its decision, the appellate incidental to any other legitimate business or activity authority mandated by Section 187 of the Insurance
court distinguished between P & I Clubs vis-à-vis of the surety; Code. It maintains a resident agent in the Philippines to
conventional insurance. The appellate court also held (c) doing any kind of business, including a
solicit insurance and to collect payments in its behalf. We
that Pioneer merely acted as a collection agent of reinsurance business, specifically recognized as
constituting the doing of an insurance business within note that Steamship Mutual even renewed its P & I Club
Steamship Mutual. cover until it was cancelled due to non-payment of the
the meaning of this Code;
(d) doing or proposing to do any business in calls. Thus, to continue doing business here, Steamship
CONTENTIONS: The parties admit that substance equivalent to any of the foregoing in a Mutual or through its agent Pioneer, must secure a
Steamship Mutual is a P & I Club. Steamship Mutual manner designed to evade the provisions of this license from the Insurance Commission.
admits it does not have a license to do business in the Code.
Philippines although Pioneer is its resident agent. This Since a contract of insurance involves public interest,
relationship is reflected in the certifications issued by the The same provision also provides, the fact that no profit regulation by the State is necessary. Thus, no insurer or
Insurance Commission. is derived from the making of insurance contracts, insurance company is allowed to engage in the
agreements or transactions, or that no separate or direct insurance business without a license or a certificate of
Petitioner insists that Steamship Mutual as a P & I Club consideration is received therefor, shall not preclude the authority from the Insurance Commission.
is engaged in the insurance business. To buttress its existence of an insurance business.
assertion, it cites the definition of a P & I Club in 2.) Pioneer is the resident agent of Steamship
Hyopsung Maritime Co., Ltd. v. Court of Appeals as "an The test to determine if a contract is an insurance Mutual as evidenced by the certificate of registration
association composed of shipowners in general who contract or not, depends on the nature of the promise, issued by the Insurance Commission. It has been
band together for the specific purpose of providing the act required to be performed, and the exact nature of licensed to do or transact insurance business by virtue of
insurance cover on a mutual basis against liabilities the agreement in the light of the occurrence, the certificate of authority issued by the same agency.
incidental to shipowning that the members incur in favor contingency, or circumstances under which the However, a Certification from the Commission states that
of third parties." It stresses that as a P & I Club, performance becomes requisite. It is not by what it is Pioneer does not have a separate license to be an
Steamship Mutual’s primary purpose is to solicit and called. agent/broker of Steamship Mutual.
provide protection and indemnity coverage and for this
purpose, it has engaged the services of Pioneer to act as
Basically, an insurance contract is a contract of Although Pioneer is already licensed as an insurance
its agent.
indemnity. In it, one undertakes for a consideration to company, it needs a separate license to act as insurance
indemnify another against loss, damage or liability agent for Steamship Mutual. Section 299 of the
Respondents contend that although Steamship Mutual is arising from an unknown or contingent event. Insurance Code clearly states:
a P & I Club, it is not engaged in the insurance business
in the Philippines. It is merely an association of vessel
In particular, a marine insurance undertakes to indemnify SEC. 299 . . .
owners who have come together to provide mutual
the assured against marine losses, such as the losses No person shall act as an insurance agent or as
protection against liabilities incidental to shipowning.
incident to a marine adventure. Section 99 of the an insurance broker in the solicitation or
Respondents aver Hyopsung is inapplicable in this case
Insurance Code enumerates the coverage of marine procurement of applications for insurance, or
insurance. receive for services in obtaining insurance, any
commission or other compensation from any judgment in favor of Julita. On appeal, the decision of the authority to investigate, petitioner is liable for claims
insurance company doing business in the trial court was affirmed but deleted all awards for made under the contract. Having assumed a
Philippines or any agent thereof, without first damages and absolved petitioner Reverente. Hence, this responsibility under the agreement, petitioner is bound to
procuring a license so to act from the petition for review raising the primary argument that a answer to the extent agreed upon. In the end, the liability
Commissioner, which must be renewed health care agreement is not an insurance contract; of the health care provider attaches once the member is
annually on the first day of January, or within six hence the “incontestability clause” under the Insurance hospitalized for the disease or injury covered by the
months thereafter. . . Code does not apply. agreement or wherever he avails of the covered benefits
which he has prepaid.
7. ISSUE 1: Whether or not the health care agreement is
G.R. No. 125678 March 18, 2002 not an insurance contract Being a contract of adhesion, the terms of an insurance
PHILAMCARE HEALTH SYSTEMS, INC., petitioner, contract are to be construed strictly against the party
Vs. COURT OF APPEALS and JULITA TRINOS, which prepared the contract – the insurer. By reason of
respondents. HELD 1: YES. Section 2 (1)of the Insurance Code the exclusive control of the insurance company over the
YNARES-SANTIAGO, J.: defines a contract of insurance as an agreement terms and phraseology of the insurance contract,
whereby one undertakes for a consideration to indemnify ambiguity must be strictly interpreted against the insurer
another against loss, damage, or liability arising from an and liberally in favor of the insured, especially to avoid
INTERNET DIGEST ONLY forfeiture. This is equally applicable to Health Care
unknown or contingent event.
Agreements.
FACTS: Ernani Trinos applied for a health care coverage Section 3 of the Insurance Code states that any
with Philamcare Health Systems, Inc. To the question contingent or unknown event, whether past or future, ISSUE 3: W/N the spouse being "not" legal wife can
‘Have you or any of your family members ever consulted which my damnify a person having an insurable against claim - YES
or been treated for high blood pressure, heart trouble, him, may be insured against. Every person has an
diabetes, cancer, liver disease, asthma or peptic ulcer?’, insurable interest in the life and health of himself.
Ernani answered ‘No’. Under the agreement, Ernani is HELD 3: Sec. 10. Every person has an insurable interest
entitled to avail of hospitalization benefits and out-patient in the life and health:
Section 10 provides that every person has an insurable (1) of himself, of his spouse and of his children;
benefits. The coverage was approved for a period of one interest in the life and health (1) of himself, of his spouse
year from March 1, 1988 to March 1, 1989. The (2) of any person on whom he depends wholly or in part
and of his children. for education or support, or in whom he has a pecuniary
agreement was however extended yearly until June 1,
1990 which increased the amount of coverage to a interest;
The insurable interest of respondent’s husband in (3) of any person under a legal obligation to him for the
maximum sum of P75,000 per disability. obtaining the health care agreement was his own health. payment of money, respecting property or service, of
The health care agreement was in the nature of non-life which death or illness might delay or prevent the
During the period of said coverage, Ernani suffered a insurance, which is primarily a contract of indemnity.
heart attack and was confined at the Manila Medical performance; and
Once the member incurs hospital, medical or any other (4) of any person upon whose life any estate or interest
Center (MMC) for one month. While in the hospital, his expense arising from sickness, injury or other stipulated
wife Julita tried to claim the benefits under the health vested in him depends.
contingent, the health care provider must pay for the
care agreement. However, the Philamcare denied her same to the extent agreed upon under the contract.
claim alleging that the agreement was void because  not the legal wife (deceased was previously
Ernani concealed his medical history. Doctors at the ISSUE 2: Whether or not there is concealment of married to another woman who was still alive)
MMC allegedly discovered at the time of Ernani’s material fact made by Ernani  health care agreement is in the nature of a
confinement that he was hypertensive, diabetic and contract of indemnity.
asthmatic, contrary to his answer in the application form. HELD 2: NO. The answer assailed by petitioner was in  payment should be made to the party who
Thus, Julita paid for all the hospitalization expenses. response to the question relating to the medical history incurred the expenses
of the applicant. This largely depends on opinion rather 8. G.R. No. 169737 February 12, 2008
After Ernani was discharged from the MMC, he was than fact, especially coming from respondent’s husband CORONA, J.:
attended by a physical therapist at home. Later, he was who was not a medical doctor. Where matters of opinion BLUE CROSS HEALTH CARE, INC., vs. NEOMI* and
admitted at the Chinese General Hospital. Due to or judgment are called for answers made I good faith and DANILO OLIVARES
financial difficulties, however, respondent brought her without intent to deceive will not avoid a policy even
husband home again. In the morning of April 13, 1990, though they are untrue.
Ernani had fever and was feeling very weak. Respondent QUICK FACTS: Neomi Olivares applied for a health care
program with Blue Cross. A month after she applied, she
was constrained to bring him back to the Chinese The fraudulent intent on the part of the insured must be suffered from a stroke. Ailments due to “pre-existing conditions”
General Hospital where he died on the same day. established to warrant rescission of the insurance were excluded from the coverage. She was confined in Medical
contract. Concealment as a defense for the health care City and discharged with a bill of P34,217.20. Blue Cross
Julita filed an action for damages and reimbursement of provider or insurer to avoid liability is an affirmative refused to pay unless she had her physician’s certification that
her expenses plus moral damages attorney’s fees defense and the duty to establish such defense by she was suffering from a pre-existing condition. When Blue
against Philamcare and its president, Dr. Benito Cross still refused to pay, she filed suit in the MTC. The health
satisfactory and convincing evidence rests upon the
Reverente. The Regional Trial court or Manila rendered care company rebutted by saying that the physician didn’t
provider or insurer. In any case, with or without the
disclose the condition due to the patient’s invocation of the Petitioner argues that respondents prevented Dr. Saniel the courts with "extreme jealousy" and "care" and with
doctor-client privilege. from submitting his report regarding the medical a "jaundiced eye." Since petitioner had the burden of
condition of Neomi. Hence, it contends that the proving exception to liability, it should have made its own
FACTS: Respondent Neomi T. Olivares applied for a presumption that evidence willfully suppressed would be assessment of whether respondent Neomi had a pre-
health care program with petitioner Blue Cross Health adverse if produced should apply in its favor. existing condition when it failed to obtain the attending
Care, Inc., a health maintenance firm. From October 16, Respondents counter that the burden was on petitioner physician's report. It could not just passively wait for Dr.
2002 to October 15, 2003, she paid the amount of to prove that Neomi's stroke was excluded from the Saniel's report to bail it out. The mere reliance on a
P11,117. She also availed of the additional service of coverage of their agreement because it was due to a disputable presumption does not meet the strict standard
limitless consultations for an additional amount of pre-existing condition. It failed to prove this. required under our jurisprudence.
P1,000. The application was approved on October 22,
2002. In the health care agreement, ailments due to "pre- ISSUE #1: Whether or not petitioner was able to prove ISSUE #2: Whether or not petitioner was liable for moral
existing conditions" were excluded from the coverage. that respondent Neomi's stroke was caused by a pre- and exemplary damages and attorney's fees
existing condition and therefore was excluded from the HELD #2: The RTC and CA found that there was a
On November 30, 2002, or barely 38 days from the coverage of the health care agreement. factual basis for the damages adjudged against
effectivity of her health insurance, respondent Neomi petitioner. They found that it was guilty of bad faith in
suffered a stroke and was admitted at the Medical City HELD #1: No. We agree with respondents. denying a claim based merely on its own perception that
which was one of the hospitals accredited by petitioner. there was a pre-existing condition.
On December 2, 2002, she was informed that she could *Please see full text for the List of Disabilities
In Philamcare Health Systems, Inc. v. CA, we ruled that considered pre-existing conditions. *
be discharged from the hospital. She incurred hospital
a health care agreement is in the nature of a non-life
expenses amounting to P34,217.20. Consequently, she
insurance. It is an established rule in insurance contracts
requested from the representative of petitioner at 9. G.R. No. 167330 (PHILIPPINE HEALTH CARE
that when their terms contain limitations on liability, they
Medical City a letter of authorization in order to settle her PROVIDERS, INC. v. CIR)
should be construed strictly against the insurer. These
medical bills. But petitioner refused to issue the letter
are contracts of adhesion the terms of which must be
and suspended payment pending the submission of a
interpreted and enforced stringently against the insurer PHILIPPINE HEALTH CARE PROVIDERS, INC., v.
certification from her attending physician that the stroke
which prepared the contract. This doctrine is equally COMMISSIONER OF INTERNAL REVENUE
she suffered was not caused by a pre-existing condition.
applicable to health care agreements. G.R. No. 167330
June 12, 2008
She was discharged from the hospital on December 3, Corona, J.
Petitioner never presented any evidence to prove that
2002. On December 5, 2002, she demanded that
respondent Neomi's stroke was due to a pre-existing
petitioner pay her medical bill. When petitioner still Note: Contention of Petitioner: its health care
condition. It merely speculated that Dr. Saniel's report
refused, she and her husband, respondent Danilo agreement is not a contract of insurance but a contract
would be adverse to Neomi, based on her invocation of
Olivares, were constrained to settle the bill. They for the provision on a prepaid basis of medical services,
the doctor-patient privilege. This was a disputable
thereafter filed a complaint for collection of sum of including medical check-up, that are not based on loss or
presumption at best.
money against petitioner in the MeTC on January 8, damage. Petitioner also insists that it is not engaged in
2003. In its answer dated January 24, 2003, petitioner
11
the insurance business. It is a health maintenance
maintained that it had not yet denied respondents' claim Section 3 (e), Rule 131 of the Rules of Court states: organization regulated by the Department of Health, not
as it was still awaiting Dr. Saniel's report. Sec. 3. Disputable presumptions. ― The following an insurance company under the jurisdiction of the
presumptions are satisfactory if uncontradicted, but may Insurance Commission. For these reasons, petitioner
be contradicted and overcome by other evidence: asserts that the health care agreement is not subject to
Dr. Saniel stated that there was patient-physician
xxx xxx xxx DST.
confidentiality and as a doctor, she should not release
(e) That evidence willfully suppressed would be adverse
any medical information concerning her neurologic status
if produced. Provision Involved: Section 185 of the 1997 Tax Code
to anyone without her approval.
Suffice it to say that this presumption does not apply if which provides:
(a) the evidence is at the disposal of both parties; (b) the
MeTC: dismissed the complaint for lack of cause of action. It suppression was not willful; (c) it is merely corroborative
held that the best person to determine whether or not the stroke Section 185. Stamp tax on fidelity bonds and
or cumulative and (d) the suppression is an exercise other insurance policies. - On all policies of insurance or
she suffered was not caused by "pre-existing conditions" is her
attending physician Dr. Saniel who treated her and conducted of a privilege. Here, respondents' refusal to present or bonds or obligations of the nature of indemnity for loss,
the test during her confinement. allow the presentation of Dr. Saniel's report was justified. damage, or liability made or renewed by any person,
RTC: reversed MeTC and ordered petitioner to pay It was privileged communication between physician and association or company or corporation transacting the
respondents.The RTC held that it was the burden of petitioner to patient. business of accident, fidelity, employer's liability, plate,
prove that the stroke of respondent Neomi was excluded from
glass, steam boiler, burglar, elevator, automatic sprinkler,
the coverage of the health care program for being caused by a
pre-existing condition. It was not able to discharge that burden. Furthermore, as already stated, limitations of liability on or other branch of insurance (except life, marine, inland,
CA: affirmed the decision of the RTC the part of the insurer or health care provider must be and fire insurance), and all bonds, undertakings, or
construed in such a way as to preclude it from evading recognizances, conditioned for the performance of the
its obligations. Accordingly, they should be scrutinized by duties of any office or position, for the doing or not doing
of anything therein specified, and on all obligations through the execution of specific instruments. In
guaranteeing the validity or legality of any bond or other particular, the DST under Section 185 of the 1997 Tax Similarly, the insurable interest of every member of
obligations issued by any province, city, municipality, or Code is imposed on the privilege of making or renewing petitioner's health care program in obtaining the health
other public body or organization, and on all obligations any policy of insurance (except life, marine, inland and care agreement is his own health. Under the agreement,
guaranteeing the title to any real estate, or guaranteeing fire insurance), bond or obligation in the nature of petitioner is bound to indemnify any member who incurs
any mercantile credits, which may be made or renewed indemnity for loss, damage, or liability. hospital, medical or any other expense arising from
by any such person, company or corporation, there shall sickness, injury or other stipulated contingency to the
be collected a documentary stamp tax of fifty centavos Under the law, a contract of insurance is an agreement extent agreed upon under the contract.
(P0.50) on each four pesos (P4.00), or fractional part whereby one undertakes for a consideration to indemnify
thereof, of the premium charged. another against loss, damage or liability arising from an Petitioner's contention that it is a health maintenance
unknown or contingent event. The event insured against organization and not an insurance company is irrelevant.
Facts: Petitioner is a domestic corporation whose must be designated in the contract and must either be Contracts between companies like petitioner and the
primary purpose is "to establish, maintain, conduct and unknown or contingent. beneficiaries under their plans are treated as insurance
operate a prepaid group practice health care delivery contracts.
system or a health maintenance organization to take Petitioner's health care agreement is primarily a contract
care of the sick and disabled persons enrolled in the of indemnity. And in the recent case of Blue Cross
health care plan and to provide for the administrative, Healthcare, Inc. v. Olivares, this Court ruled that a health
legal, and financial responsibilities of the organization." care agreement is in the nature of a non-life insurance
Individuals enrolled in its health care programs pay an policy.
annual membership fee and are entitled to various 10. G.R. No. 112329 (VIRGINIA A. PEREZ v. COURT
preventive, diagnostic and curative medical services. Contrary to petitioner's claim, its health care agreement OF APPEALS and BF LIFEMAN INSURANCE
is not a contract for the provision of medical services. CORPORATION)
The deficiency DST assessment was imposed on Petitioner does not actually provide medical or hospital
petitioner's health care agreement with the members of services but merely arranges for the same and pays for
its health care program pursuant to Section 185 of the them up to the stipulated maximum amount of coverage.
FACTS: Primitivo B. Perez had been insured with the BF
1997 Tax Code. It is also incorrect to say that the health care agreement
Lifeman Insurance Corporation since 1980 for
is not based on loss or damage because, under the said
P20,000.00. Sometime in October 1987, an agent of the
Petitioner protested the assessment in a letter. As agreement, petitioner assumes the liability and
insurance corporation, Rodolfo Lalog, visited Perez in
respondent did not act on the protest, petitioner filed a indemnifies its member for hospital, medical and related
Guinayangan, Quezon and convinced him to apply for
petition for review in the Court of Tax Appeals seeking expenses.
additional insurance coverage of P50,000.00, to avail of
the cancellation of the deficiency VAT and DST
the ongoing promotional discount of P400.00 if the
assessments. Furthermore, the fact that petitioner must relieve its
premium were paid annually.
The CTA rendered a decision, where the Petition is member from liability by paying for expenses arising from
Partially Granted. the stipulated contingencies belies its claim that its
services are prepaid. The expenses to be incurred by On October 20, 1987, Primitivo B. Perez accomplished
Respondent appealed the CTA decision to the CA. He each member cannot be predicted beforehand, if they an application form for the additional insurance coverage
claimed that petitioner's health care agreement was a can be predicted at all. Petitioner assumes the risk of of P50,000.00. On the same day, petitioner Virginia A.
contract of insurance subject to DST under Section 185 paying for the costs of the services even if they are Perez, Primitivos wife, paid P2,075.00 to Lalog. The
of the 1997 Tax Code. significantly and substantially more than what the receipt issued by Lalog indicated the amount received
member has "prepaid." This is insurance. was a "deposit.”] Unfortunately, Lalog lost the application
The CA rendered its decision. It held that petitioner's form accomplished by Perez and so on October 28,
health care agreement was in the nature of a non-life Petitioner's health care agreement is substantially similar 1987, he asked the latter to fill up another application
insurance contract subject to DST. to that involved in Philamcare Health Systems, Inc. v. form. On November 1, 1987, Perez was made to
CA. This Court ruled in Philamcare Health Systems, Inc.: undergo the required medical examination, which he
Petitioner moved for reconsideration but the CA denied passed.
it. Hence, this petition.
The insurable interest of the subscriber Pursuant to the established procedure of the company,
Issue: Whether the contract is a health care agreement in obtaining the health care agreement was his Lalog forwarded the application for additional insurance
in the nature of an insurance contract and therefore own health. The health care agreement was in of Perez, together with all its supporting papers, to the
subject to the documentary stamp tax (DST) imposed the nature of non-life insurance, which is office of BF Lifeman Insurance Corporation at Gumaca,
under Section 185 of Republic Act 8424 Tax Code of primarily a contract of indemnity. Once the Quezon which office was supposed to forward the
1997? member incurs hospital, medical or any other papers to the Manila office.
expense arising from sickness, injury or other
Held: Yes, the DST is levied on the exercise by persons stipulated contingency, the health care provider On November 25, 1987, Perez died in an accident. He
of certain privileges conferred by law for the creation, must pay for the same to the extent agreed was riding in a banca which capsized during a storm. At
revision, or termination of specific legal relationships upon under the contract.
the time of his death, his application papers for the 11. G. R. No. L-2294 (FILIPINAS COMPANIA DE Held: YES. The Philippine Insurance Law (Act No. 2427,
additional insurance of P50,000.00 were still with the SEGUROS v. CHRISTERN, HUENEFELD & CO., INC.) as amended,) in section 8, provides that "anyone except
Gumaca office. Lalog testified that when he went to a public enemy may be insured." It stands to reason that
follow up the papers, he found them still in the Gumaca an insurance policy ceases to be allowable as soon as
office and so he personally brought the papers to the an insured becomes a public enemy.
G.R. No. L-2294 May 25, 1951
Manila office of BF Lifeman Insurance Corporation. It
was only on November 27, 1987 that said papers were There is no question that majority of the stockholders of
received in Manila. Without knowing that Perez died on FILIPINAS COMPAÑIA DE SEGUROS, petitioner,
the respondent corporation were German subjects. This
November 25, 1987, BF Lifeman Insurance Corporation vs.
being so, said respondent became an enemy corporation
approved the application and issued the corresponding CHRISTERN, HUENEFELD and CO., INC.,
upon the outbreak of the war between the United States
policy for the P50,000.00 on December 2, 1987. Virginia respondent.
and Germany.
went to Manila to claim the benefits and was given
40,000.00 php. Then she claimed additional for the Facts: Respondent Christern obtained from Filipinas a
coverage under the 50,000.00 plan. The partial payment The respondent having become an enemy corporation
fire insurance policy of P1000,000, covering
made was refunded and the contract was rescinded by on December 10, 1941, the insurance policy issued in its
merchandise contained in a building located at Binondo.
BF Lifeman Insurance Corp. The lower court ruled in favor on October 1, 1941, by the petitioner (a Philippine
During the Japanese military occupation, the building
favor of Perez but the CA reversed the decision. corporation) had ceased to be valid and enforcible, and
and insured merchandise were burned. The respondent
since the insured goods were burned after December 10,
its claim under the policy. The total loss suffered by the
1941, and during the war, the respondent was not
respondent was fixed at P92,650.
entitled to any indemnity under said policy from the
ISSUE: Was there a perfected insurance contract? petitioner.
The petitioner refused to pay the claim on the ground
RULING: None. The contract was not perfected. As a that the policy in favor of the respondent had ceased to
However, elementary rules of justice (in the absence of
contract, consent, object, cause must be complied with. be in force on the date the U.S. declared war on
specific provision in the Insurance Law) require that the
Germany with the respondent Corporation being
premium paid by the respondent for the period covered
controlled by German subjects and the petitioner being a
The assent of private respondent BF Lifeman Insurance by its policy from December 11, 1941, should be
company under American jurisdiction (though organized
Corporation therefore was not given when it merely returned by the petitioner.
by Philippine laws) when the policy was issued on
received the application form and all the requisite October 1, 1941. The petitioner, however, paid to the
supporting papers of the applicant. Its assent was given respondent the sum of P92,650 on April 19, 1943 under 12. G.R. NO. 114427 FEBRUARY 6, 1995
when it issues a corresponding policy to the applicant. orders from the military government.
Under the abovementioned provision, it is only when the
ARMANDO GEAGONIA v. COURT OF APPEALS and
applicant pays the premium and receives and accepts
The insurer filed for a suit to recover the sum. The COUNTRY BANKERS INSURANCE CORPORATION
the policy while he is in good health that the contract
of insurance is deemed to have been perfected. contention was that the policy ceased to be effective
because of the outbreak of the war and that the payment FACTS: Armando Geagonia, the owner of Norman's
made by the petitioner to the respondent corporation Mart, obtained from Country Bankers Insurance
It is not disputed, however, that when Primitivo died on during the Japanese military occupation was under Corporation (Country Bankers) fire insurance policy for
November 25, 1987, his application papers for additional pressure. P100,000.00. The 1 year period policy covered Stock-in-
insurance coverage were still with the branch office of
trade consisting principally of dry goods. The said policy
respondent corporation in Gumaca and it was only two
The trial and the appellate courts dismissed the action. contained the following condition: 3. The insured shall
days later, or on November 27, 1987, when Lalog
give notice to the Company of any insurance or
personally delivered the application papers to the head
insurances already affected, or which may subsequently
office in Manila. The Court of Appeals overruled the contention of the
be effected, covering any of the property or properties
petitioner that the respondent corporation became an
consisting of stocks in trade, goods in process and/or
Consequently, there was absolutely no way the enemy when the United States declared war against
inventories only hereby insured, and unless such notice
acceptance of the application could have been Germany, relying on English and American cases which
be given and the particulars of such insurance or
communicated to the applicant for the latter to accept held that a corporation is a citizen of the country or state
insurances be stated therein or endorsed in this policy
inasmuch as the applicant at the time was already by and under the laws of which it was created or
pursuant to Section 50 of the Insurance Code, by or on
dead. In the case of Enriquez vs. Sun Life Assurance organized. It rejected the theory that nationality of private
behalf of the Company before the occurrence of any loss
Co. of Canada, recovery on the life insurance of the corporation is determine by the character or citizenship
or damage, all benefits under this policy shall be deemed
deceased was disallowed on the ground that the contract of its controlling stockholders.
forfeited, provided however, that this condition shall not
for annuity was not perfected since it had not been apply when the total insurance or insurances in force at
proved satisfactorily that the acceptance of the Issue: Whether or not the insurance policy became the time of the loss or damage is not more than
application ever reached the knowledge of the applicant. ineffective upon the declaration of the war? P200,000.00.
On 27 May 1990, fire of accidental origin broke out thereof shall avoid it, otherwise the breach of an insurance and thus avert the perpetration of fraud. When
destroying Geogonia’s insured stock-in-trade. This immaterial provision does not avoid the policy." Such a a property owner obtains insurance policies from two or
prompted him to file with Country Bankers a claim under condition is a provision which invariably appears in fire more insurers in a total amount that exceeds the
the policy. The latter denied the claim because it found insurance policies and is intended to prevent an increase property's value, the insured may have an inducement to
that at the time of the loss Geogonia's stocks-in-trade in the moral hazard. It is commonly known as the destroy the property for the purpose of collecting the
were likewise covered by fire insurance policies for additional or "other insurance" clause and has been insurance. The public as well as the insurer is interested
P100,000.00 each, issued by the Cebu Branch of the upheld as valid and as a warranty that no other in preventing a situation in which a fire would be
Philippines First Insurance Co., Inc. (PFIC). The basis of insurance exists. Its violation would thus avoid the policy. profitable to the insured.
the Country Banker's denial was Geogonia's alleged However, in order to constitute a violation, the other
violation of Condition 3 of the policy. insurance must be upon same subject matter, the same
interest therein, and the same risk.
13. Spouses NILO CHA and STELLA UY CHA, and
Geogonia then filed a complaint for the recovery of UNITED INSURANCE CO., INC., petitioners, vs.
P100,000.00 under fire insurance policy. In a letter, he (b) NO. It is a cardinal rule on insurance that a policy or COURT OF APPEALS and CKS DEVELOPMENT
admitted that at the time he obtained the Country insurance contract is to be interpreted liberally in favor of CORPORATION, respondents.
Banker's fire insurance policy he knew that the two the insured and strictly against the company, the reason G.R. No. 124520. August 18, 1997
policies issued by the PFIC were already in existence; being, undoubtedly, to afford the greatest protection
however, he had no knowledge of the provision in the which the insured was endeavoring to secure when he
FACTS: Petitioner-spouses Nilo Cha and Stella Uy-Cha,
Country Banker's policy requiring him to inform it of the applied for insurance. It is also a cardinal principle of law
as lessees, entered into a lease contract with private
prior policies; this requirement was not mentioned to him that forfeitures are not favored and that any construction
respondent CKS Development Corporation, as lessor, on
by the their agent; and had it been mentioned, he would which would result in the forfeiture of the policy benefits
5 October 1988.
not have withheld such information. He further asserted for the person claiming thereunder, will be avoided, if it is
that the total of the amounts claimed under the three possible to construe the policy in a manner which would
policies was below the actual value of his stocks at the permit recovery, as, for example, by finding a waiver for One of the stipulations of the one (1) year lease contract
time of loss, which was P1,000,000.00. In its answer, such forfeiture. Stated differently, provisions, conditions states:
Country Bankers specifically denied the allegations in the or exceptions in policies which tend to work a forfeiture
complaint and set up as its principal defense the violation of insurance policies should be construed most strictly 18. x x x. The LESSEE shall not insure against fire the
of Condition 3 of the policy. against those for whose benefits they are inserted, and chattels, merchandise, textiles, goods and effects placed
most favorably toward those against whom they are at any stall or store or space in the leased premises
The Insurance Commission found that the petitioner did intended to operate. The reason for this is that, except without first obtaining the written consent and approval of
not violate Condition 3 as he had no knowledge of the for riders which may later be inserted, the insured sees the LESSOR. If the LESSEE obtain(s) the insurance
existence of the two fire insurance policies obtained from the contract already in its final form and has had no voice thereof without the consent of the LESSOR then the
the PFIC. This was reversed by the Court of Appeals in the selection or arrangement of the words employed policy is deemed assigned and transferred to the
therein. On the other hand, the language of the contract LESSOR for its own benefit; x x x
was carefully chosen and deliberated upon by experts
ISSUE: (a) whether the petitioner had prior knowledge of and legal advisers who had acted exclusively in the
the two insurance policies issued by the PFIC when he Notwithstanding the above stipulation in the lease
interest of the insurers and the technical language
obtained the fire insurance policy from the private contract, the Cha spouses insured against loss by fire
employed therein is rarely understood by ordinary
respondent, thereby, for not disclosing such fact, their merchandise inside the leased premises for Five
laymen.
violating Condition 3 of the policy; and (b) if he had, Hundred Thousand (P500,000.00) with the United
whether he is precluded from recovering therefrom. Insurance Co., Inc. (without the written consent of private
With these principles in mind, we are of the opinion that respondents CKS.
Condition 3 of the subject policy is not totally free from
HELD: ambiguity and must, perforce, be meticulously analyzed.
(a) YES. Geogonia knew of the prior policies issued by On the day that the lease contract was to expire, fire
Such analysis leads us to conclude that (a) the
the PFIC. His letter to Country Bankers conclusively broke out inside the leased premises.When CKS learned
prohibition applies only to double insurance, and (b) the
proves this knowledge. His testimony to the contrary of the insurance earlier procured by the Cha spouses
nullity of the policy shall only be to the extent exceeding
before the Insurance Commissioner and which the latter (without its consent), it wrote the insurer (United) a
P200,000.00 of the total policies obtained.
relied upon cannot prevail over a written admission made demand letter asking that the proceeds of the insurance
ante litem motam. It was, indeed, incredible that he did contract (between the Cha spouses and United) be paid
not know about the prior policies since these policies Furthermore, by stating within Condition 3 itself that such directly to CKS, based on its lease contract with Cha
were not new or original. condition shall not apply if the total insurance in force at spouses.
the time of loss does not exceed P200,000.00, the
private respondent was amenable to assume a co-
Condition 3 is a condition which is not proscribed by United refused to pay CKS. Hence, the latter filed a
insurer's liability up to a loss not exceeding P200,000.00.
law. Its incorporation in the policy is allowed by Section complaint against the Cha spouses and United.
What it had in mind was to discourage over-insurance.
75 of the Insurance Code which provides that "[a] policy Indeed, the rationale behind the incorporation of "other
may declare that a violation of specified provisions insurance" clause in fire policies is to prevent over-
RTC: ordered defendant United to pay CKS the amount inside the leased premises under the provisions of should not be held liable because it was destroyed due
of P335,063.11 and defendant Cha spouses to pay Section 17 of the Insurance Code which provide. to fortuities event or force majeure
P50,000.00 as exemplary damages, P20,000.00 as
attorney’s fees and costs of suit. Section 17. The measure of an insurable - RTC held that IMC and LSPI retained ownership of the
interest in property is the extent to which the delivered goods until fully paid, it must bear the loss (res
CA: affirmed the trial court decision, deleting however insured might be damnified by loss of injury perit domino) while the CA: Reversed - sales invoices is
the awards for exemplary damages and attorney’s fees. thereof." an exception under Article 1504 (1) of the Civil Code to
res perit domino
Therefore, respondent CKS cannot, under the
ISSUE: Insurance Code a special law be validly a beneficiary ISSUE: WON Insurance Company of North America can
Whether or not the paragraph 18 of the lease contract of the fire insurance policy taken by the petitioner- claim against Gaisano Cagayan for the debt that was
entered into between CKS and the Cha spouses is valid spouses over their merchandise. This insurable insured
insofar as it provides that any fire insurance policy interest over said merchandise remains with the insured,
obtained by the lessee (Cha spouses) over their the Cha spouses. The automatic assignment of the HELD: YES. It was partly granted and the order to pay
merchandise inside the leased premises is deemed policy to CKS under the provision of the lease contract P535,613 is DELETED. The Insurance policy is clear
assigned or transferred to the lessor (CKS) if said policy previously quoted is void for being contrary to law and/or that the subject of the insurance is the book debts and
is obtained without the prior written of the latter. public policy. The proceeds of the fire insurance policy NOT goods sold and delivered to the customers and
thus rightfully belong to the spouses Nilo Cha and Stella dealers of the insured
Uy-Cha (herein co-petitioners). The insurer (United)
HELD:
cannot be compelled to pay the proceeds of the fire Under ART. 1504. Unless otherwise agreed, the goods
NO. It is, of course, basic in the law on contracts that the
insurance policy to a person (CKS) who has no remain at the seller's risk until the ownership therein is
stipulations contained in a contract cannot be contrary to
insurable interest in the property insured. transferred to the buyer, but when the ownership therein
law, morals, good customs, public order or public policy.
is transferred to the buyer the goods are at the buyer's
The liability of the Cha spouses to CKS for violating their risk whether actual delivery has been made or not,
Sec. 18 of the Insurance Code provides:
lease contract in that Cha spouses obtained a fire except that:
Sec. 18. No contract or policy of insurance on
insurance policy over their own merchandise, without the
property shall be enforceable except for the
consent of CKS, is a separate and distinct issue which (1) Where delivery of the goods has been made to
benefit of some person having an insurable
we do not resolve in this case. the buyer or to a bailee for the buyer, in pursuance
interest in the property insured.
14. GAISANO CAGAYAN, INC. vs. INSURANCE of the contract and the ownership in the goods has
COMPANY OF NORTH AMERICA been retained by the seller merely to secure
A non-life insurance policy such as the fire G.R. No. 147839 June 8, 2006 performance by the buyer of his obligations under
insurance policy taken by petitioner-spouses over
the contract, the goods are at the buyer's risk from
their merchandise is primarily a contract of
AUSTRIA-MARTINEZ, J.: the time of such delivery;
indemnity. Insurable interest in the property insured
Here, IMC and LSPI did not lose complete interest over
must exist at the time the insurance takes effect and
FACTS: Intercapitol Marketing Corporation (IMC) is the the goods. They have an insurable interest until full
at the time the loss occurs. The basis of such
maker of Wrangler Blue Jeans. while Levi Strauss payment of the value of the delivered goods. Unlike the
requirement of insurable interest in property insured is
(Phils.) Inc. (LSPI) is the local distributor of products civil law concept of res perit domino, where ownership is
based on sound public policy: to prevent a person from
bearing trademarks owned by Levi Strauss & Co. IMC the basis for consideration of who bears the risk of loss,
taking out an insurance policy on property upon which he
and LSPI separately obtained from Insurance Company in property insurance, one's interest is not determined by
has no insurable interest and collecting the proceeds of
of North America fire insurance policies for their book concept of title, but whether insured has substantial
said policy in case of loss of the property. In such a case,
debt endorsements related to their ready-made clothing economic interest in the property
the contract of insurance is a mere wager which is void
under Section 25 of the Insurance Code, which provides: materials which have been sold or delivered to various
customers and dealers of the Insured anywhere in the Under Section 13 of our Insurance Code defines
Philippines which are unpaid 45 days after the time of insurable interest as "every interest in property,
SECTION 25. Every stipulation in a policy of
the loss. On February 25, 1991, Gaisano Superstore whether real or personal, or any relation thereto, or
Insurance for the payment of loss, whether
Complex in Cagayan de Oro City, owned by Gaisano liability in respect thereof, of such nature that a
the person insured has or has not any interest
Cagayan, Inc., containing the ready-made clothing contemplated peril might directly damnify the
in the property insured, or that the policy shall
materials sold and delivered by IMC and LSPI was insured." Parenthetically, under Section 14 of the
be received as proof of such interest, and
consumed by fire and On February 4, 1992 Insurance same Code, an insurable interest in property may
every policy executed by way of gaming or
Company of North America filed a complaint for consist in: (a) an existing interest; (b) an inchoate
wagering, is void.
damages against Gaisano Cagayan, Inc. alleges that interest founded on existing interest; or (c) an
IMC and LSPI filed their claims under their respective fire expectancy, coupled with an existing interest in that
In the present case, it cannot be denied that CKS has insurance policies which it paid thus it was subrogated to out of which the expectancy arises. Anyone has an
no insurable interest in the goods and merchandise their rights. Gaisano Cagayan, Inc alleged that they insurable interest in property who derives a benefit
from its existence or would suffer loss from its 1975, Tan Lee Siong died of hepatoma. Hence, The "incontestability clause" added by the second
destruction. It is sufficient that the insured is so petitioners filed with respondent company their claim for paragraph of Section 48 is in force for two years.
situated with reference to the property that he would the proceeds of the life insurance policy. After this, the defenses of concealment or
be liable to loss should it be injured or destroyed by misrepresentation no longer lie.
the peril against which it is insured an insurable However, the insurance company denied the said claim
interest in property does not necessarily imply a and rescinded the policy by reason of the alleged The petitioners argue that no evidence was
property interest in, or a lien upon, or possession of, misrepresentation and concealment of material facts presented to show that the medical terms were
the subject matter of the insurance, and neither the made by the deceased Tan Lee Siong in his application explained in a layman's language to the insured.
title nor a beneficial interest is requisite to the for insurance. The premiums paid on the policy were They also argue that no evidence was presented by
existence of such an interest insurance in this case thereupon refunded. The petitioners contend that the respondent company to show that the questions
is not for loss of goods by fire but for petitioner's respondent company no longer had the right to rescind appearing in Part II of the application for insurance
accounts with IMC and LSPI that remained unpaid the contract of insurance as rescission must allegedly be were asked, explained to and understood by the
45 days after the fire the obligation is pecuniary in done during the lifetime of the insured within two years deceased so as to prove concealment on his part.
nature obligor should be held exempt from liability and prior to the commencement of action. This couldn’t be accepted because the insured
when the loss occurs thru a fortuitous event only signed the form. He affirmed the correctness of all
holds true when the obligation consists in the the entries.
delivery of a determinate thing and there is no The Court of Appeals dismissed ' the petitioners' appeal
stipulation holding him liable even in case of from the Insurance Commissioner's decision for lack of
fortuitous event. merit. The company records show that the deceased was
examined by Dr. Victoriano Lim and was found to be
ISSUE: WON the Philippine American Life Insurance diabetic and hypertensive. He was also found to
Under Article 1263 of the Civil Code in an obligation have suffered from hepatoma. Because of the
to deliver a generic thing, the loss or destruction of Company didn’t have the right to rescind the contract of
insurance as rescission must allegedly be done during concealment made by the deceased, the company
anything of the same kind does not extinguish the was thus misled into accepting the risk and
obligation (Genus nunquan perit) The subrogation the lifetime of the insured within two years and prior to
the commencement of action. approving his application as medically fit.
receipt, by itself, is sufficient to establish not only the
relationship of respondent as insurer and IMC as the
insured, but also the amount paid to settle the HELD: No. Petition dismissed. The Insurance Code 16. VIOLETA R. LALICAN vs. THE INSULAR LIFE
insurance claim. states in Section 48:“Whenever a right to rescind a ASSURANCE COMPANY LIMITED, AS
contract of insurance is given to the insurer by any REPRESENTED BY THE PRESIDENT VICENTER
provision of this chapter, such right must be exercised AVILON, G.R. No. 183526, 25 August 2009
Under Art. 2207. If the plaintiff's property has been
insured, and he has received indemnity from the previous to the commencement of an action on the
insurance company for the injury or loss arising out contract. FACTS:
of the wrong or breach of contract complained of, the During his lifetime Eulogio Lalican, husband of
insurance company shall be subrogated to the rights After a policy of life insurance made payable on the Violeta Lalican herein petitioner, applied for insurance
of the insured against the wrongdoer or the person death of the insured shall have been in force during the policy with INSURANCE LIFE , through agent named
who has violated the contract. lifetime of the insured for a period of two years from the GASPAN MALALUAN, which contained a 20-Year
Here, as to LSPI, no subrogation receipt was offered date of its issue or of its last reinstatement, the insurer Endowment Variable Income Package Flexi Plan worth
in evidence. cannot prove that the policy is void ab initio or is P500,000.00,[6] with two riders valued at P500,000.00
rescindable by reason of the fraudulent concealment or each.[7] Thus, the value of the policy amounted to
misrepresentation of the insured or his agent.” P1,500,000.00. Violeta was named as the primary
Failure to substantiate the claim of subrogation is beneficiary.
fatal to petitioner's case for recovery of the amount
of P535,613. The so-called "incontestability clause" in the second
15. G.R. No. 48049 June 29, 1989 paragraph prevents the insurer from raising the defenses Under the terms of the policy Eulogio was to
EMILIO TAN, JUANITO TAN, ALBERTO TAN and of false representations insofar as health and previous pay the premiums on a quarterly basis in the amount of
ARTURO TAN, Petitioners, vs. THE COURT OF diseases are concerned if the insurance has been in P8,062.00, payable every 24 April, 24 July, 24 October
APPEALS and THE PHILIPPINE AMERICAN LIFE force for at least two years during the insured's lifetime. and 24 January of each year, until the end of the 20-year
INSURANCE COMPANY, Respondents. Ponente: period of the policy. According to the Policy Contract,
GUTIERREZ, JR., J. there was a grace period of 31 days for the payment of
The policy was in force for a period of only one year each premium subsequent to the first. If any premium
and five months. Considering that the insured died was not paid on or before the due date, the policy would
FACTS: Tan Lee Siong, father of herein petitioners, before the two-year period had lapsed, respondent be in default, and if the premium remained unpaid until
applied for life insurance in the amount of P80,000.00 company is not, therefore, barred from proving that the end of the grace period, the policy would
with respondent company Philippine American Life the policy is void ab initio by reason of the insured's automatically lapse and become void.
Insurance Company. Said application was approved and fraudulent concealment or misrepresentation.
a corresponding policy was issued effective November 5,
1973, with petitioners as the beneficiaries. On April 26,
Eulogio paid the premiums due on 24 July 1997 To reinstate a policy means to restore the same application for reinstatement. Examining the said
and 24 October 1997. However, he failed to pay the to premium-paying status after it has been permitted to provisions, the court finds the same clearly written in
premium due on 24 January 1998, even after the lapse lapse.[39] Both the Policy Contract and the Application terms that are simple enough to admit of only one
of the grace period of 31 days. Policy No. 9011992, for Reinstatement provide for specific conditions for the interpretation. They are clearly not ambiguous,
therefore, lapsed and became void. reinstatement of a lapsed policy. equivocal or uncertain that would need further
construction. The same are written on the very face of
On 26 May 1998, an Application for 2. Yes, However, An insurable interest is one the application just above the space where [Eulogio]
Reinstatement which was not processed because the of the most basic and essential requirements in an signed his name. It is inconceivable that he signed it
interest was not paid. On 17 September 1998, Eulogio insurance contract. In general, an insurable interest is without reading and understanding its import.
went to Malaluans house and submitted a second that interest which a person is deemed to have in the
Application for Reinstatement] of Policy No. 9011992, subject matter insured, where he has a relation or
including the amount of P17,500.00, representing connection with or concern in it, such that the person will
payments for the overdue interest on the premium for 24 derive pecuniary benefit or advantage from the
January 1998, and the premiums which became due on preservation of the subject matter insured and will suffer
24 April 1998 and 24 July 1998. pecuniary loss or damage from its destruction,
termination, or injury by the happening of the event 17. NEW LIFE ENTERPRISES and JULIAN SY vs
A while later, on the same day, 17 September insured against. The existence of an insurable interest HON. COURT OF APPEALS, EQUITABLE
1998, Eulogio died of cardio-respiratory arrest gives a person the legal right to insure the subject matter INSURANCE CORPORATION, RELIANCE SURETY
secondary to electrocution. of the policy of insurance.[36] Section 10 of the AND INSURANCE CO., INC. and WESTERN
Insurance Code indeed provides that every person has GUARANTY CORPORATION, G.R. No. 94071 March
an insurable interest in his own life. Section 19 of the 31, 1992
On 28 September 1998, Violeta filed with same code also states that an interest in the life or health
Insular Life a claim for payment of the full proceeds of of a person insured must exist when the insurance takes
Policy No. 9011992 which was not granted because the Facts: New Life Enterprises, owned by Julian Sy and
effect, but need not exist thereafter or when the loss
reinstatement application was not granted. Hence Violeta Jose Sy Bang, engaged in the sale of construction
occurs.
filed a Complaint for Death Claim Benefit in the RTC. materials at Iyam, Lucena City.
RTC dismissed the action, hence this petition for review
on certiorari. Upon more extensive study of the Petition, it
Julian Sy insured the stocks in trade of New Life
becomes evident that the matter of insurable interest is
Enterprises with Western Guaranty Corporation,
entirely irrelevant in the case at bar. It is actually beyond
Reliance Surety and Insurance Co. Inc., and Equitable
question that while Eulogio was still alive, he had an
ISSUE: 1. WON Violeta is entitled to the insurance Insurance Corporation, all sister companies.
insurable interest in his own life, which he did insure
benefit. under Policy No. 9011992. The real point of contention
2. WON EULOGIO has insurable interest. herein is whether Eulogio was able to reinstate the Western Guaranty Corporation issued Fire Insurance
lapsed insurance policy on his life before his death on 17 Policy No. 37201 in the amount of P350,000.00.
HELD: September 1998. Reliance Surety and Insurance Co., Inc. issued Fire
Insurance Policy No. 69135 in the amount of
1. No. That Policy No. 9011992 had already P300,000.00. Equitable Insurance Corporation issued
__________________________________
lapsed is a fact beyond dispute. Eulogios filing of his first Fire Insurance Policy No. 39328 in the amount of
The New Lexicon Websters Dictionary defines
Application for Reinstatement with Insular Life, through P200,000.00.
ambiguity as the quality of having more than one
Malaluan, on 26 May 1998, constitutes an admission that meaning and an idea, statement or expression capable
Policy No. 9011992 had lapsed by then. Insular Life did of being understood in more than one sense. In Nacu vs. The building occupied by the New Life Enterprises was
not act on Eulogios first Application for Reinstatement, Court of Appeals, 231 SCRA 237 (1994), the Supreme gutted by fire at about 2:00 o'clock in the morning of
since the amount Eulogio simultaneously deposited was Court stated that[:] October 19, 1982, the stocks in trade inside said building
sufficient to cover only the P8,062.00 overdue premium were insured against fire in the total amount of
for 24 January 1998, but not the P322.48 overdue P1,550,000.00.
interests thereon. On 17 September 1998, Eulogio
submitted a second Application for Reinstatement to Any ambiguity in a contract, whose terms are susceptible
of different interpretations as a result thereby, must be Ultimately, the three insurance companies denied
Insular Life, again through Malaluan, depositing at the
read and construed against the party who drafted it on plaintiffs' claim for payment from the insurance policy.
same time P17,500.00, to cover payment for the overdue
interest on the premium for 24 January 1998, and the the assumption that it could have been avoided by the
premiums that had also become due on 24 April 1998 exercise of a little care. Because of the denial of their claims for payment by the
and 24 July 1998. On the very same day, Eulogio three (3) insurance companies, petitioner filed separate
passed away. civil actions against the former before the Regional Trial
Court, which was granted in favor of petitioner New Life.
In the instant case, the dispute arises from the afore-
quoted provisions written on the face of the second
Respondent Court of Appeals reversed said judgment of the same insurance claims adjuster. Availment of the
the trial court. services of the same agents and adjusters by different Manulife Philippines, Inc. (Manulife) instituted a
companies is a common practice in the insurance Complaint for Rescission of Insurance Contracts against
Condition No. 3 of said insurance policies, otherwise business and such facts do not warrant the speculative Hermenegilda Ybañez (Hermenegilda) and the BPI
known as the "Other Insurance Clause," is uniformly conclusion of the trial court. Family Savings Bank (BPI Family). It is alleged in the
contained in all the aforestated insurance contracts of Complaint that Insurance which Manulife issued in favor
herein petitioners, as follows: Whether or not petitioner’s claim was filed out of of Dr. Gumersindo Solidum Ybañez (insured), were void
time? YES. due to concealment or misrepresentation of material
"3. The insured shall give notice to the facts in the latter's applications for life insurance; that
Company of any insurance or insurances already Policy Condition No. 27 of their insurance contract with Hermenegilda, wife of the said insured, was revocably
effected, or which may subsequently be effected, Reliance provides: designated as beneficiary in the subject insurance
covering any of the property or properties consisting policies; that on November 17, 2003, when one of the
of stocks in trade, goods in process and/or inventories subject insurance policies had been in force for only one
only hereby insured, and unless such notice be given "27. Action or suit clause. - If a claim be made and
year and three months, while the other for only four
and the particulars of such insurance or insurances rejected and an action or suit be not commenced
either in the Insurance Commission or any court of months, the insured died; that Manulife conducted an
be stated therein or endorsed on this policy pursuant
to Section 50 of the Insurance Code, by or on behalf competent jurisdiction of notice of such rejection, or in investigation into the circumstances leading to the said
of the Company before the occurrence of any loss or case of arbitration taking place as provided herein, insured's death, in view of the aforementioned entries in
damage, all benefits under this policy shall be within twelve (12) months after due notice of the the said insured's Death Certificate; that Manulife
deemed forfeited, provided however, that this award made by the arbitrator or abitrators or umpire, thereafter concluded that the insured misrepresented or
condition shall not apply when the total insurance or then the claim shall for all purposes be deemed to concealed material facts at the time the subject
insurances in force at the time of loss or damage is have been abandoned and shall not thereafter be insurance policies were applied for; and that for this
not more than P200,000.00." recoverable hereunder."
reason Manulife accordingly denied Hermenegilda's
death claims and refunded the premiums that the insured
Issue: Whether or not petitioner violated the “Other The condition contained in an insurance policy that paid on the subject insurance policies.
Insurance Clause” of the insurance contracts? claims must be presented within one year after rejection
is not merely a procedural requirement but an important
Due to the Insured's concealment of material facts at the
Held: YES. The terms of the contract are clear and matter essential to a prompt settlement of claims against
time the subject insurance policies were applied for and
insurance companies as it demands that insurance suits
unambiguous. The insured is specifically required to issued, [Manulife] exercised its right to rescind the
be brought by the insured while the evidence as to the
disclose to the insurer any other insurance and its subject insurance contracts and denied the claims on
origin and cause of destruction have not yet
particulars which he may have effected on the same those policies. Manulife thus prayed that judgment be
disappeared.
subject matter. The knowledge of such insurance by the rendered finding its act of rescinding the subject
insurer's agents, even assuming the acquisition thereof insurance policies proper; declaring these subject
by the former, is not the "notice" that would estop the In the case at hand, there was a considerable lapse of insurance policies null and void; and discharging. it from
insurers from denying the claim. time from their receipt of the insurer's clarificatory letter any obligation whatsoever under these policies.
dated March 30, 1983, up to the time the complaint was
filed in court on January 31, 1984. The one-year
Furthermore, when the words and language of Hermenegilda countered that Manulife's own insurance
prescriptive period was yet to expire on November 29,
documents are clear and plain or readily understandable agent, Ms. Elvira Monteclaros herself] assured [the
1983 or about eight (8) months from the receipt of the
by an ordinary reader thereof, there is absolutely no insured,] that there would be no problem regarding the
clarificatory letter, but petitioners let the period lapse
room for interpretation or construction anymore. application for the insurance policy. In tact, it was
without bringing their action in court. We accordingly find
Monteclaros who filled up everything in the questionnaire
no "peculiar circumstances" sufficient to relax the
Additionally, it is and was incumbent upon petitioner Sy (Annex "C" of the [C]omplaint), so that [all that the
enforcement of the one-year prescriptive period and we,
to read the insurance contracts, and this can be insured needed to do was sign it,] and it's done. [It was
therefore, hold that petitioners' claim was definitely filed
reasonably expected of him considering that he has also Ms. Monteclaros who herself] checked in advance
out of time.
been a businessman. Unfortunately, he inexplicably all the boxes in Annex "C," [that the insured himself was
18. MANULIFE PHILIPPINES, INC. vs
claimed that he had not read the terms of the policies. required to answer or check].
HERMENEGILDA YBANEZ, G.R. No. 204736,
November 28, 2016
Whether or not there was notice to the other [Manulife] accepted [the insured's] application, and now
companies due to the fact that the policies were that a claim for the benefits [is] made, [Manulife now]
[ G.R. No. 204736, November 28, 2016 ]
obtained from the same agent? NO. says that [the insured] misrepresented and concealed his
MANULIFE PHILIPPINES, INC.,[1] PETITIONER, VS.
past illnesses[!] In the form filled up by [Dr. Winifredo F.
HERMENEGILDA YBAÑEZ, RESPONDENT.
Lumapas,] Manulife's [company] physician, dated
The conclusion of the trial court that Reliance and D E C I S I O N, DEL CASTILLO, J.:
9/10/02, [the insured] checked the column which says
Equitable are "sister companies" is an unfounded ''yes" (to] the following questions:
conjecture drawn from the mere fact that Yap Kam FACTS:
Chuan was an agent for both companies which also had
 Have you had electrocardiograms, when, why, confirmatory of the findings of fact of the RTC, as in this application that he had undergone lithotripsy due to
result? ([Manulife's company physician] wrote case, are conclusive upon this Court. kidney stone in 1987, but was discharged after three
the answer which stated that result was days, and no recurrence claimed.
normal.) Thus, this Court must defer to the findings of fact of the
RTC - as affirmed or confirmed by the CA - that On February 5, 2001, Sun Life approved Atty. Jesus Jr.’s
Manulife's Complaint for rescission of the insurance application and issued an insurance policy. The policy
 Have you seen a doctor, or had treatment policies in question was totally bereft of factual and legal indicated that his beneficiaries are entitled to death
operation on hospital case during the last five bases because it had utterly failed to prove that the benefits should he die on or before February 5, 2021 or a
years? insured had committed the alleged misrepresentation/s certain sum of money if Atty. Jesus is still living on the
or concealment/s of material facts imputed against him. endowment date.
Hermegegilda further claimed that at the time when both
insurance policies in question were submitted for The RTC correctly held that the CDH’s medical records On May 11, 2001, Atty. Jesus Jr. died as a result of a
approval to [Manulife, the latter had had all the that might have established the insured’s purported gunshot wound in San Joaquin, Iloilo. As such, Ma.
forewarnings that should have put it on guard or on misrepresentation/s or concealment/s was inadmissible Daisy filed a Claimant’s Statement with Sun Life to seek
notice that things were not what it wanted them to be, for being hearsay, given the fact that Manulife failed to the death benefits indicated in his insurance policy.
reason enough to bestir it into exercising greater present the physician or any responsible official of the
prudence and caution to further inquire into) the health or CDH who could confirm or attest to the due execution Sun Life denied the claim on the ground that the details
medical history of [the insured]. In particular, Manulife and authenticity of the alleged medical records. on Atty. Jesus Jr.’s medical history were not disclosed in
ought to have noted the fact that the insured was at that his application. Simultaneously, Sun Life tendered a
time already 65 years old, x x x that he had a previous check representing the refund of the premiums paid by
Manulife's sole witness gave no evidence at all relative to
operation, and x x x that his health was "below average. Atty. Jesus Jr. It later filed a Complaint for Rescission
the particulars of the purported concealment or
misrepresentation allegedly perpetrated by the insured. before the RTC and prayed for judicial confirmation of
Manulife presented its sole witness in the person of Ms. In fact, Victoriano merely perfunctorily identified the Atty. Jesus Jr.’s rescission of insurance policy.
Jessiebelle Victoriano (Victoriano ), the Senior Manager documentary exhibits adduced by Manulife; she never
of its Claims and Settlements Department. The oral testified in regard to the circumstances attending the According to Sun Life, the undisclosed fact suggested
testimony of this witness chiefly involved identifying execution of these documentary exhibits much less in that the insured was in “renal failure” and at a high risk
herself as the Senior Manager of Manulife's Claims and regard to its contents. Of course, the mere mechanical medical condition. Consequently, had it known such fact,
Settlements Department and also identifying the act of identifying these documentary exhibits, without the it would not have issued the insurance policy in favor of
evidence. After due proceedings, the RTC dismissed testimonies of the actual participating parties thereto, Atty. Jesus Jr.11
Manulife's Complaint. The RTC found no merit at all in adds up to nothing. These documentary exhibits did not
Manulife's Complaint for rescission of the subject automatically validate or explain themselves. RTC: Sun Life violated Sections 241, paragraph 1(b), (d),
insurance policies because it utterly failed to prove that
and (e) and 242 of the Insurance Code when it refused
the insured had committed the alleged
"The fraudulent intent on the part of the insured to pay the rightful claim of the respondents. It held that
misrepresentation/s or concealment/s. The CA affirmed
must be established to entitle the insurer to rescind Atty. Jesus Jr. did not commit material concealment and
the decision of RTC.
the contract. Misrepresentation as a defense of the misrepresentation when he applied for life insurance with
insurer to avoid liability is an affirmative defense and Sun Life. It observed that given the disclosures and the
ISSUE: the duty to establish such defense by satisfactory waiver and authorization to investigate executed by Atty.
and convincing evidence rests upon the insurer." Jesus Jr. to Sun Life, the latter had all the means of
Whether the CA committed any reversible error in For failure of Manulife to prove intent to defraud on ascertaining the facts allegedly concealed by the
affirming the RTC Decision dismissing Manulife's the part of the insured, it cannot validly sue for applicant.
Complaint for rescission of insurance contracts for failure rescission of insurance contracts.
to prove concealment on the part of the insured. CA: Affirmed the RTC decision.

HELD: 19. G.R. No. 211212, June 8, 2016 Issue: Whether there was misrepresentation on the part
SUN LIFE OF CANADA (PHILIPPINES), INC. vs MA, of Atty. Jesus during his insurance application, hence, he
NO. The present recourse essentially challenges anew DAISY S. SIBYA, JESUS MANUEL S. SIBY A Ⅲ. is not entitled to the insurance claim.
the findings of fact by both the RTC and the CA that the JAIME LUIS S. SIBYA, and The Estate of the
Complaint for rescission of the insurance policies in deceased ATTY. JESUS SIBYA JR., Ruling:
question will not prosper because Manulife failed to No.
prove concealment on the part of the insured. This is Facts:
not allowed. It is horn-book law that in appeal by On January 10, 2001, Atty. Jesus Sibya, Jr. (Atty. Jesus In Manila Bankers Life Insurance Corporation v.
certiorari to this Court under Rule 45 of the Revised Jr.) applied for life insurance with Sun Life. In his Aban,22 the Court held that if the insured dies within the
Rules of Court, the findings of fact by the CA especially Application for Insurance, he indicated that he had two-year contestability period, the insurer is bound to
where such findings of fact are affirmatory or sought advice for kidney problems. He indicated in his
make good its obligation under the policy, regardless of because the two-year contestability period had not
the presence or lack of concealment or yet lapsed inasmuch as the insurance policy was
misrepresentation. The Court held: reinstated only on December 27, 1999, whereas Felipe
died on September 22, 2001
Section 48 serves a noble purpose, as it regulates [ Sec. 48. Of the Insurance Code,
the actions of both the insurer and the insured. Whenever a right to rescind a contract of
Under the provision, an insurer is given two years — insurance is given to the insurer by any
provision of this chapter, such right must be
from the effectivity of a life insurance contract and 20. THE INSULAR LIFE ASSURANCE COMPANY, LTD. exercised previous to the commencement
while the insured is alive — to discover or prove that vs. PAZ Y. KHU, FELIPE Y. KHU, JR., and FREDERICK of an action on the contract.
the policy is void ab initio or is rescindible by reason Y. KHU, G.R. No. 195176, April 18, 2016 After a policy of life insurance made
of the fraudulent concealment or misrepresentation G.R. No. 195176. April 18, 2016. *
payable on the death of the insured shall
of the insured or his agent. After the two-year have been in force during the lifetime of the
period lapses, or when the insured dies within insured for a period of two years from the
THE INSULAR LIFE ASSURANCE COMPANY, LTD., date of its issue or of its last reinstatement,
the period, the insurer must make good on the
petitioner, the insurer cannot prove that the policy is
policy, even though the policy was obtained by
vs. void ab initio or is rescindible by reason of
fraud, concealment, or misrepresentation. This is
PAZ Y. KHU, FELIPE Y. KHU, JR., and FREDERICK Y. the fraudulent concealment or
not to say that insurance fraud must be rewarded, misrepresentation of the insured or his
KHU, respondents.
but that insurers who recklessly and indiscriminately agent.]
solicit and obtain business must be penalized, for
such recklessness and lack of discrimination FACTS: On March 6, 1997, Felipe N. Khu, Sr. (Felipe)
applied for a life insurance policy with Insular Life Respondents’ Arguments: the phrase “effective June
ultimately work to the detriment of bona fide takers
under the latter's Diamond Jubilee Insurance Plan. 22, 1999” found in both the Letter of Acceptance and
of insurance and the public in general.
Felipe accomplished the required medical questionnaire in the Endorsement is unclear whether it refers to the
wherein he did not declare any illness or adverse subject of the sentence, i.e., the “reinstatement of
In the present case, Sun Life issued Atty. Jesus Jr.’s this policy” or to the subsequent phrase “changes
medical condition. Insular Life thereafter issued him
policy on February 5, 2001. Thus, it has two years from are made on the policy”; that granting that there was
Policy Number A000015683 with a face value of PI
its issuance, to investigate and verify whether the policy any obscurity or ambiguity in the insurance policy,
million. This took effect on June 22, 1997.
was obtained by fraud, concealment, or the same should be laid at the door of Insular Life as
misrepresentation. Upon the death of Atty. Jesus Jr., it was this insurance company that prepared the
however, on May 11, 2001, or a mere three months from On June 23, 1999, Felipe's policy lapsed due to non- necessary documents that make up the same
the issuance of the policy, Sun Life loses its right to payment of the premium covering the period from June
rescind the policy. As discussed in Manila Bankers, the 22, 1999 to June 23, 2000
ISSUE: Whether Felipe's reinstated life insurance policy
death of the insured within the two-year period will
render the right of the insurer to rescind the policy is already incontestable at the time of his death.
On September 7, 1999, Felipe applied for the
nugatory. As such, the incontestability period will now reinstatement of his policy and paid P25,020.00 as
set in. premium HELD: Yes. It was more than two years had lapsed from
the time the subject insurance policy was reinstated on
Assuming, however, for the sake of argument, that the June 22, 1999 vis-à-vis Felipe’s death on September 22,
On October 12, 1999, Insular Life advised Felipe that his 2001. As such, the subject insurance policy has already
incontestability period has not yet set in, the Court
application for reinstatement may only be considered if become incontestable at the time of Felipe’s death.
agrees, nonetheless, with the CA when it held that Sun
he agreed to certain conditions such as payment of
Life failed to show that Atty. Jesus Jr. committed
additional premium and the cancellation of the riders
concealment and misrepresentation. In Lalican v. The Insular Life Assurance Company,
pertaining to premium waiver and accidental death
benefits. Felipe agreed to these conditions[8] and on Limited, 597 SCRA 159 (2009), which coincidentally also
Indeed, the intent to defraud on the part of the December 27, 1999 paid the agreed additional premium involves the herein petitioner, it was there held that the
insured must be ascertained to merit rescission of of P3,054.50 reinstatement of the insured’s policy is to be reckoned
the insurance contract. Concealment as a defense for from the date when the application was processed and
the insurer to avoid liability is an affirmative defense and approved by the insurer. There, we stressed that:
On September 22, 2001, Felipe died
the duty to establish such defense by satisfactory and
convincing evidence rests upon the provider or insurer. To reinstate a policy means to restore the same
In the present case, Sun Life failed to clearly and On October 5, 2001, Paz Y. Khu, Felipe Y. Khu, Jr. .and to premium-paying status after it has been permitted to
satisfactorily establish its allegations, and is therefore Frederick Y. Khu (collectively, Felipe's beneficiaries or lapse. x x x x x x x
liable to pay the proceeds of the insurance. respondents) filed with Insular Life a claim for benefit
under the reinstated policy. This claim was denied.
In the instant case, Eulogio’s death rendered
impossible full compliance with the conditions for
Petitioner’s Arguments: respondents should not be reinstatement of Policy No. 9011992. True, Eulogio,
allowed to recover on the reinstated insurance policy before his death, managed to file his Application for
Reinstatement and deposit the amount for payment of in such a way as to preclude the insurer from Petitioner Virginia Perez went to Manila to claim the
his overdue premiums and interests thereon with noncompliance with its obligations. benefits under the insurance policies of the deceased.
Malaluan; but Policy No. 9011992 could only be She was paid P40,000.00 under the first insurance
considered reinstated after the Application for policy for P20,000.00 (double indemnity in case of
Reinstatement had been processed and approved by accident) but the insurance company refused to pay the
Insular Life during Eulogio’s lifetime and good health. claim under the additional policy coverage of
Thus, it is settled that the reinstatement of an insurance 21. VIRGINIA A. PEREZ vs. COURT OF APPEALS and P50,000.00, the proceeds of which amount to
policy should be reckoned from the date when the same BF LIFEMAN INSURANCE CORPORATION, G.R No. P150,000.00 in view of a triple indemnity rider on the
was approved by the insurer. 112329, 28 January 2000 insurance policy. The insurance company maintained
YNARES-SANTIAGO, J.: that the insurance for P50,000.00 had not been
it is settled that the reinstatement of an insurance policy perfected at the time of the death of Primitivo Perez.
should be reckoned from the date when the same was Doctrine: A contract of insurance, like all other Consequently, the insurance company refunded the
approved by the insurer. In this case, the parties differ contracts, must be assented to by both parties, either in amount of P2,075.00 which Virginia Perez had paid.
as to when the reinstatement was actually approved. person or through their agents and so long as an
Insular Life claims that it approved the reinstatement only application for insurance has not been either accepted or On September 21, 1990, private respondent BF Lifeman
on December 27, 1999. On the other hand, respondents rejected, it is merely a proposal or an offer to make a Insurance Corporation filed a complaint against Virginia
contend that it was on June 22, 1999 that the contract. A. Perez seeking the rescission and declaration of nullity
reinstatement took effect. of the insurance contract in question. Virginia A. Perez
Facts: averred that the deceased had fulfilled all his prestations
The resolution of this issue hinges on the following under the contract and all the elements of a valid
documents: contract are present.
Primitivo B. Perez had been insured with the BF Lifeman
1) Letter of Acceptance; and Insurance Corporation for P20,000.00. Agent Rodolfo
2) the Endorsement. Lalog, convinced him to apply for additional insurance RTC: Ruled in favor of petitioner; held that the premium
coverage of P50,000.00, to avail of the ongoing for the additional insurance of P50,000.00 had been fully
The Letter of Acceptance wherein Felipe affixed his promotional discount of P400.00 if the premium were paid and even if the sum of P2,075.00 were to be
signature was actually drafted and prepared by Insular paid annually. Primitivo accomplished an application considered merely as partial payment, the same does
Life. After Felipe accomplished this form, Insular Life, form for the additional insurance coverage of not affect the validity of the policy. The trial court further
through its Regional Administrative Manager, Jesse P50,000.00. On the same day, petitioner Virginia A. stated that the deceased had fully complied with the
James R. Toyhorada, issued an Endorsement dated 33 Perez, Primitivo’s wife, paid P2,075.00 to Lalog. The requirements of the insurance company. He paid, signed
January 7, 2000. receipt issued by Lalog indicated the amount received the application form and passed the medical
was a "deposit." Unfortunately, Lalog lost the application examination. He should not be made to suffer the
form accomplished by Perez and so asked the latter to subsequent delay in the transmittal of his application
This finding must be upheld not only because it form to private respondents head office since these were
fill up another application form.
accords with the evidence, but also because this is no longer within his control.
favorable to the insured who was not responsible for
causing the ambiguity or obscurity in the insurance Lalog forwarded the application for additional insurance
contract. of Perez, together with all its supporting papers, to the CA: Reversed the decision of the RTC; The insurance
office of BF Lifeman Insurance Corporation at Gumaca, contract for P50,000.00 could not have been perfected
Quezon which office was supposed to forward the since at the time that the policy was issued, Primitivo
It must be remembered that an insurance contract is a was already dead.
papers to the Manila office.
contract of adhesion which must be construed liberally in
favor of the insured and strictly against the insurer in
order to safeguard the latter’s interest. Thus, in Malayan On November 25, 1987, Perez died in an accident. At Issue: Whether there was a perfected Insurance
Insurance Corporation v. Court of Appeals, this Court the time of his death, his application papers for the Contract
held that: additional insurance of P50,000.00 were still with the
Indemnity and liability insurance policies are Gumaca office. So, Lalog he personally brought the Held: NO.
construed in accordance with the general rule of papers to the Manila office of BF Lifeman Insurance
resolving any ambiguity therein in favor of the insured, Corporation. It was only on November 27, 1987 that said
Insurance is a contract whereby, for a stipulated
where the contract or policy is prepared by the insurer. A papers were received in Manila.
consideration, one party undertakes to compensate the
contract of insurance, being a contract of adhesion, par other for loss on a specified subject by specified perils. A
excellence, any ambiguity therein should be resolved Without knowing that Perez died on November 25, contract, on the other hand, is a meeting of the minds
against the insurer; in other words, it should be 1987, BF Lifeman Insurance Corporation approved the between two persons whereby one binds himself, with
construed liberally in favor of the insured and strictly application and issued the corresponding policy for the respect to the other to give something or to render some
against the insurer. Limitations of liability should be P50,000.00 on December 2, 1987. service. Under Article 1318 of the Civil Code, there is no
regarded with extreme jealousy and must be construed Nc contract unless the following requisites concur:
1. Consent of the contracting parties; void. Article 1182 of the New Civil Code states: When after it was received. The processing of applications by
2. Object certain which is the subject matter of the the fulfillment of the condition depends upon the sole will respondent corporation normally takes two to three
contract; of the debtor, the conditional obligation shall be void. weeks, the longest being a month. In this case, however,
3. Cause of the obligation which is established. the requisite medical examination was undergone by the
In the case at bar, the following conditions were imposed deceased on November 1, 1987; the application papers
Consent must be manifested by the meeting of the offer by the respondent company for the perfection of the were forwarded to the head office on November 27,
and the acceptance upon the thing and the cause which contract of insurance: 1987; and the policy was issued on December 2, 1987.
are to constitute the contract. The offer must be certain (a).......a policy must have been issued; Under these circumstances, we hold that the delay
and the acceptance absolute. (b).......the premiums paid; and could not be deemed unreasonable so as to
(c).......the policy must have been delivered to and constitute gross negligence.
When Primitivo filed an application for insurance, paid accepted by the applicant while he is in good health.
P2,075.00 and submitted the results of his medical
examination, his application was subject to the The condition imposed by the corporation that the policy As to the recission of the Insurance Policy 056300
acceptance of private respondent BF Lifeman Insurance must have been delivered to and accepted by the
Corporation. The perfection of the contract of insurance applicant while he is in good health can hardly be SC: True, rescission presupposes the existence of a
between the deceased and respondent corporation was considered as a potestative or facultative condition. On valid contract. A contract which is null and void is no
further conditioned upon compliance with the following the contrary, the health of the applicant at the time of the contract at all and hence could not be the subject of
requisites stated in the application form: delivery of the policy is beyond the control or will of the rescission.
insurance company. Rather, the condition is a 22. GULF RESORTS, INC. vs. PHILIPPINE CHARTER
"there shall be no contract of insurance unless and until suspensive one whereby the acquisition of rights INSURANCE CORPORATION, G.R. No. 156167, May
a policy is issued on this application and that the said depends upon the happening of an event which 16, 2005
policy shall not take effect until the premium has been constitutes the condition. In this case, the suspensive Kareen
paid and the policy delivered to and accepted by me/us condition was the policy must have been delivered and FACTS:
in person while I/We, am/are in good health." accepted by the applicant while he is in good health.  Gulf Resorts, Inc at Agoo, La Union was
There was non-fulfillment of the condition, however, insured with American Home Assurance
inasmuch as the applicant was already dead at the time Company which includes loss or damage
The assent of private respondent BF Lifeman Insurance the policy was issued. Hence, the non-fulfillment of the
Corporation therefore was not given when it merely to shock to any of the property insured by
condition resulted in the non-perfection of the contract. this Policy occasioned by or through or in
received the application form and all the requisite Sdaa mi
supporting papers of the applicant. Its assent was given consequence of earthquake
so  July 16, 1990: an earthquake struck
when it issues a corresponding policy to the applicant. As stated above, a contract of insurance, like other
Under the abovementioned provision, it is only when Central Luzon and Northern Luzon so the
contracts, must be assented to by both parties either
the applicant pays the premium and receives and properties and 2 swimming pools in its
in person or by their agents. So long as an
accepts the policy while he is in good health that the Agoo Playa Resort were damaged
application for insurance has not been either
contract of insurance is deemed to have been  August 23, 1990: Gulf's claim was denied
accepted or rejected, it is merely an offer or proposal
perfected. on the ground that its insurance policy only
to make a contract. The contract, to be binding from the
afforded earthquake shock coverage to the
date of application, must have been a completed
two swimming pools of the resort
contract, one that leaves nothing to be done, nothing to
be completed, nothing to be passed upon, or
 Petitioner contends that pursuant
to this rider, no qualifications were
determined, before it shall take effect. There can be no
In the case of Enriquez vs. Sun Life Assurance Co. of placed on the scope of the
contract of insurance unless the minds of the parties
Canada, recovery on the life insurance of the deceased earthquake shock
have met in agreement.
was disallowed on the ground that the contract for coverage. Thus, the policy
annuity was not perfected since it had not been proved extended earthquake shock
satisfactorily that the acceptance of the application ever Prescinding from the foregoing, respondent coverage to all of the insured
reached the knowledge of the applicant. corporation cannot be held liable for gross properties.
negligence. It should be noted that an application is  RTC: Favored American Home -
a mere offer which requires the overt act of the endorsement rider means that only the two
Contention: Petitioner insists that the condition imposed insurer for it to ripen into a contract. Delay in acting
by respondent corporation that a policy must have been swimming pools were insured against
on the application does not constitute acceptance even earthquake shock
delivered to and accepted by the proposed insured in though the insured has forwarded his first premium with
good health is potestative being dependent upon the will  CA: affirmed RTC
his application. The corporation may not be penalized ISSUE: W/N Gulf can claim for its properties aside from
of the corporation and is therefore null and void. for the delay in the processing of the application the 2 swimming pools
papers. Moreover, while it may have taken some time for
SC: A potestative condition depends upon the exclusive the application papers to reach the main office, in the
will of one of the parties. For this reason, it is considered case at bar, the same was acted upon less than a week HELD: YES. Affirmed.
 It is basic that all the provisions of the the office of the defendant only on April 15, 1963. At any rate, it is not disputed that petitioner paid in full all
insurance policy should be examined and PACIFIC claimed for insurance to the value of the premiums as called for by the statement issued by
interpreted in consonance with each other. P19,286.79. Woodmen’s requested an adjustment private respondent after the issuance of the two regular
 All its parts are reflective of the company (First Philippine Adjustment Corporation) to marine insurance policies, thereby leaving no account
true intent of the parties. assess the damage. It submitted its report, where it unpaid by petitioner due on the insurance coverage,
found that the loss of 30 pieces of logs is not covered by which must be deemed to include the Cover Note. If the
Policies Nos. 53 HO 1032 and 1033 but within the Note is to be treated as a separate policy instead of
1,250,000 bd. ft. covered by Cover Note 1010 insured for integrating it to the regular policies subsequently issued,
Insurance Code $70,000.00. the purpose and function of the Cover Note would be set
at naught or rendered meaningless, for it is in a real
Section 2(1) The adjustment company submitted a computation of the sense a contract, not a mere application for insurance
WORKMEN’S probable liability on the loss sustained by which is a mere offer.
contract of insurance as an agreement whereby one undertakes forthe
a consideration to indemnify
shipment, in the another
total amount against loss, damage or liability arising from an unknown or contingent event
of P11,042.04.
The non-payment of premium on the Cover Note is,
WORKMENS wrote PACIFIC denying the latter's claim therefore, no cause for the petitioner to lose what is due
 An insurance premium is the consideration on the ground they defendant's investigation revealed it as if there had been payment of premium, for non-
paid an insurer for undertaking to that the entire shipment of logs covered by the two payment by it was not chargeable against its fault. Had
indemnify the insured against a specified marine policies were received in good order at their point all the logs been lost during the loading operations, but
peril. of destination. It was further stated that the said loss may after the issuance of the Cover Note, liability on the note
 In the subject policy, no premium be considered as covered under Cover Note No. 1010 would have already arisen even before payment of
payments were made with regard because the said Note had become null and void by premium. This is how the cover note as a “binder” should
to earthquake shock coverage, virtue of the issuance of Marine Policy Nos. 53 HO 1032 legally operate; otherwise, it would serve no practical
except on the two swimming and 1033. purpose in the realm of commerce, and is supported by
pools. The denial of the claim by the defendant was brought by the doctrine that where a policy is delivered without
 Ana si maam sa duha ka swimming pool lang the plaintiff to the attention of the Insurance requiring payment of the premium, the presumption is
liable Commissioner. The Insurance Commissioner ruled in that a credit was intended and policy is valid.
23. favor of indemnifying Pacific Timber. The company
G.R. No. L-38613 February 25, 1982 added that the cover note is null and void for lack of ISSUE #2: Whether or not the Insurance company was
DE CASTRO, ** J.: valuable consideration. The trial court ruled in petitioner’s absolved from responsibility due to unreasonable delay
PACIFIC TIMBER EXPORT CORPORATION, petitioner, favor while the CA dismissed the case. Hence this in giving notice of loss.
Vs. THE HONORABLE COURT OF APPEALS and appeal.
WORKMEN'S INSURANCE COMPANY, INC., HELD #2: Delay of insured in reporting the loss must be
respondents. ISSUE #1: Whether or not the cover note was null and objected to promptly by insurer. Sending of insurance
void for lack of valuable consideration adjuster to assess the loss amounts to waiver of delay in
FACTS: The Pacific Timber Export Corporation secured giving notice of loss.
temporary insurance from the Workmen's Insurance HELD #1: A “Cover Note” issued in advance of the  The defense of delay as raised by private
Company, Inc. for its exportation of 1,250,000 board feet issuance of a marine policy is binding as an insurance respondent in resisting the claim cannot be
of Philippine Lauan and Apitong logs to be shipped from contract although no separate premium was paid sustained. The law requires this ground of delay
Quezon Province to Okinawa and Tokyo, Japan. therefor. to be promptly and specifically asserted when a
Workmen’s Insurance issued a cover note insuring the  The fact that no separate premium was paid on claim on the insurance agreement is made. The
cargo of the plaintiff subject to its terms and conditions. the Cover Note before the loss insured against undisputed facts show that instead of invoking
After the issuance of the cover note, but before the occurred, does not militate against the validity the ground of delay in objecting to petitioner’s
issuance of the two marine policies, some of the logs of petitioner’s contention, for no such premium claim of recovery on the cover note, it took
intended to be exported were lost during loading could have been paid, since by the nature of the steps clearly indicative that this particular
operations in the Diapitan Bay. Cover Note, it did not contain, as all Cover ground for objection to the claim was never in
Notes do not contain particulars of the shipment its mind. The nature of this specific ground for
While the logs were alongside the vessel, bad weather that would serve as basis for the computation of resisting a claim places the insurer on duty to
developed resulting in 75 pieces of logs which were the premiums. As a logical consequence, no inquire when the loss took place, so that it could
rafted together co break loose from each other. 45 separate premiums are intended or required to determine whether delay would be a valid
pieces of logs were salvaged, but 30 pieces were verified be paid on a Cover Note. This is a fact admitted ground upon which to object to a claim against
to have been lost or washed away as a result of the by an official of respondent company, Juan it.
accident. Pacific Timber informed Workmen’s about the Jose Camacho, in charge of issuing cover notes
loss of 32 pieces of logs during loading of SS woodlock. of the respondent company In the proceedings that took place later in the Office of
the Insurance Commissioner, private respondent should
Although dated April 4, 1963, the letter was received in then have raised this ground of delay to avoid liability. It
did not do so. It must be because it did not find any recommending the approval of the 20-year endowment an authorized insurance agent of Pacific Life at Cebu
delay, as this Court fails to find a real and substantial insurance plan to children. branch office, is indubitably aware that said company
sign thereof. But even on the assumption that there was does not offer the life insurance applied for. When he
delay, this Court is satisfied and convinced that as Thereafter, Helen Go died of influenza with complication filed the insurance application in dispute, private
expressly provided by law, waiver can successfully be of bronchopneumonia. Thereupon, private respondent respondent was, therefore, only taking the chance that
raised against private respondent. sought the payment of the proceeds of the insurance, but Pacific Life will approve the recommendation of
24. GREAT PACIFIC LIFE ASSURANCE COMPANY vs. having failed in his effort, he filed the action for the Mondragon for the acceptance and approval of the
HONORABLE COURT OF APPEALS, G.R. No. L- 31845 recovery of the same before the Court of First Instance application in question along with his proposal that the
April 30, 1979 of Cebu. insurance company starts to offer the 20-year
Janine endowment insurance plan for children less than seven
years. Nonetheless, the record discloses that Pacific Life
ISSUE: Whether the binding deposit receipt constituted a had rejected the proposal and recommendation.
GREAT PACIFIC LIFE ASSURANCE COMPANY v. temporary contract of the life insurance in question?
HONORABLE COURT OF APPEALS Secondly, having an insurable interest on the life of his
G.R. No. L-31845 one-year old daughter, aside from being an insurance
April 30, 1979 HELD: NO, the binding deposit receipt in question is agent and an offense associate of petitioner Mondragon,
De Castro, J. merely an acknowledgment, on behalf of the company, private respondent Ngo Hing must have known and
that the latter's branch office had received from the followed the progress on the processing of such
applicant the insurance premium and had accepted the application and could not pretend ignorance of the
NOTE: Conditions by the company to approve life application subject for processing by the insurance Company's rejection of the 20-year endowment life
insurance: company; and that the latter will either approve or reject insurance application.
The provisions printed on Exhibit E show that the binding the same on the basis of whether or not the applicant is
deposit receipt is intended to be merely a provisional or "insurable on standard rates." Since petitioner Pacific
temporary insurance contract and only upon compliance Life disapproved the insurance application of respondent
of the following conditions: (1) that the company shall be Ngo Hing, the binding deposit receipt in question had
satisfied that the applicant was insurable on standard never become in force at any time. 25. MELECIO COQUIA, MARIA ESPANUEVA and
rates; (2) that if the company does not accept the MANILA YELLOW TAXICAB CO., INC. vs. FIELDMEN'S
application and offers to issue a policy for a different INSURANCE CO., INC., G.R. No. L-23276 November
plan, the insurance contract shall not be binding until the The binding deposit receipt is, merely conditional and
does not insure outright. As held by this Court, where an 29, 1968
applicant accepts the policy offered; otherwise, the Terry
deposit shall be refunded; and (3) that if the applicant is agreement is made between the applicant and the agent,
not able according to the standard rates, and the no liability shall attach until the principal approves the
company disapproves the application, the insurance risk and a receipt is given by the agent. The acceptance WEB DIGEST.
applied for shall not be in force at any time, and the is merely conditional and is subordinated to the act of the
premium paid shall be returned to the applicant. company in approving or rejecting the application. Thus,
in life insurance, a "binding slip" or "binding receipt" does
not insure by itself. FACTS: On Dec. 1, 1961, Fieldmen’s Insurance co.
FACTS: On March 14, 1957, private respondent Ngo Issued in favor of the Manila Yellow Taxicab a common
Hing filed an application with the Great Pacific Life carrier insurance policy with a stipulation that the
Assurance Company for a twenty-year endownment As held in De Lim vs. Sun Life Assurance Company of company shall indemnify the insured of the sums which
policy on the life of his one-year old daughter Helen Go. Canada, supra, "a contract of insurance, like other the latter wmy be held liable for with respect to “death or
Mondragon type-wrote the data on the application form contracts, must be assented to by both parties either in bodily injury to any fair paying passenger including the
which was signed by private respondent Ngo Hing. The person or by their agents ... The contract, to be binding driver and conductor”.
latter paid the annual premium going over to the from the date of the application, must have been a
Company, but he retained the amount of P1,317.00 as completed contract, one that leaves nothing to be done,
nothing to be completed, nothing to be passed upon, or The policy also stated that in “the event of the death of
his commission for being a duly authorized agent of the driver, the Company shall indemnify his personal
Pacific Life. Then on April 30, 1957, Mondragon received determined, before it shall take effect. There can be no
contract of insurance unless the minds of the parties representatives and at the Company’s option may make
a letter from Pacific Life disapproving the insurance indemnity payable directly to the claimants or heirs of the
application. The letter stated that the said life insurance have met in agreement."
claimants.”
application for 20-year endowment plan is not available
for minors below seven years old, but Pacific Life can Other relevant facts: No meeting of the minds:
consider the same under the Juvenile Triple Action Plan. We are not impressed with private respondent's During the policy’s lifetime, a taxicab of the insured
contention that failure of petitioner Mondragon to driven by Coquia met an accident and Coquia died.
communicate to him the rejection of the insurance When the company refused to pay the only heirs of
The non-acceptance of the insurance plan by Pacific Life Coquia, his parents, they institued this complaint. The
was allegedly not communicated by petitioner application would not have any adverse effect on the
allegedly perfected temporary contract. In this first place, company contends that plaintiffs have no cause of action
Mondragon to private respondent Ngo Hing. Instead, since the Coquias have no contractual relationship with
Mondragon wrote back Pacific Life again strongly there was no contract perfected between the parties who
had no meeting of their minds. Private respondent, being the company.
membership in the group life insurance plan. In his indebtedness shall first be paid to the creditor and the
ISSUE: Whether or not plaintiffs have the right to collect insurance application, he answered that he was in good balance of sum assured, if there is any, shall then be
on the policy. health and that he had not consulted a doctor or any of paid to the beneficiary/ies designated by the debtor.
the enumerated ailments, including hypertension. On When DBP submitted the insurance claim against
August 6, 1984, Dr. Leuterio died due to massive Grepalife, the latter denied payment thereof, interposing
RULING: YES. cerebral hemorrhage. Consequently, DBP submitted a the defense of concealment committed by the insured.
death claim to Grepalife. Grepalife denied the claim Thereafter, DBP collected the debt from the mortgagor
Athough, in general, only parties to a contract may bring alleging that Dr. Leuterio was not physically healthy and took the necessary action of foreclosure on the
an action based thereon, this rule is subject to when he applied for an insurance coverage. Grepalife residential lot of Dr. Leuterio. In Gonzales La O vs. Yek
exceptions, one of which is found in the second insisted that Dr. Leuterio did not disclose he had been Tong Lin Fire & Marine Ins. Co. we held: Insured, being
paragraph of Article 1311 of the Civil Code of the suffering from hypertension, which caused his death. the person with whom the contract was made, is
Philippines, reading: "If a contract should contain some Allegedly, such non-disclosure constituted concealment primarily the proper person to bring suit thereon. * * *
stipulation in favor of a third person, he may demand its that justified the denial of the claim. Subject to some exceptions, insured may thus sue,
fulfillment provided he communicated his acceptance to although the policy is taken wholly or in part for the
the obligor before its revocation. A mere incidental On October 20, 1986, the widow of the late Dr. benefit of another person named or unnamed, and
benefit or interest of a person is not sufficient. The Leuterio, Medarda V. Leuterio, filed a complaint against although it is expressly made payable to another as his
contracting parties must have clearly and deliberately Grepalife. During the trial, Dr. Hernando Mejia, who interest may appear or otherwise. * * * Although a policy
conferred a favor upon a third person." This is but the issued the death certificate, was called to testify. Dr. issued to a mortgagor is taken out for the benefit of the
restatement of a well-known principle concerning Mejias findings, based partly from the information given mortgagee and is made payable to him, yet the
contracts pour autrui, the enforcement of which may be by Medarda, stated that Dr. Leuterio complained of mortgagor may sue thereon in his own name, especially
demanded by a third party for whose benefit it was headaches presumably due to high blood pressure. The where the mortgagees interest is less than the full
made, although not a party to the contract, before the inference was not conclusive because Dr. Leuterio was amount recoverable under the policy.
stipulation in his favor has been revoked by the not autopsied, hence, other causes were not ruled out.
contracting parties Indeed, Insured may be regarded as the real party in
The RTC ruled in favor of Medarda and held interest, although he has assigned the policy for the
Grepalife liable to pay DBP. This was sustained by the purpose of collection, or has assigned as collateral
In the case at bar, the policy under consideration is CA. security any judgment he may obtain.
typical of contracts pour autrui this character being made
more manifest by the fact that the deceased driver paid 2. YES, Grepalife should be held liable. It merely relied
fifty percent (50%) of the corresponding premiums, which ISSUE: 1. WON DBP has insurable interest as creditor on the testimony of Dr. Hernando Mejia, as supported
were deducted from his weekly commissions. Under by the information given by the widow of the decedent.
these conditions, it is clear that the Coquias — who, 2. WON Grepalife should be held liable (this refers to an The medical findings were not conclusive because Dr.
admittedly, are the sole heirs of the deceased — have a alleged concealment that Grepalife interposed as its Mejia did not conduct an autopsy on the body of the
direct cause of action against the Company, and, since defense to annul the insurance contract). decedent. Furthermore, there was no sufficient proof that
they could have maintained this action by themselves, the insured had suffered from hypertension. Aside from
without the assistance of the insured it goes without the statement of the insured’s widow who was not even
saying that they could and did properly join the latter in HELD: 1. YES, DBP has insurable interest as sure if the medicines taken by Dr. Leuterio were for
filing the complaint herein. creditor. Section 8 of the Insurance Code provides: hypertension, Grepalife had not proven nor produced
Unless the policy provides, where a mortgagor of any witness who could attest to Dr. Leuterios medical
property effects insurance in his own name providing that history. Since Grepalife had failed to establish that there
the loss shall be payable to the mortgagee, or assigns a was concealment made by the insured, it cannot refuse
26.G.R. No. 113899 OCTOBER 13, 1999 policy of insurance to a mortgagee, the insurance is payment of the claim.
deemed to be upon the interest of the mortgagor, who
does not cease to be a party to the original contract, and The fraudulent intent on the part of the insured
GREAT PACIFIC LIFE ASSURANCE CORP. V. CA any act of his, prior to the loss, which would otherwise must be established to entitle the insurer to rescind the
avoid the insurance, will have the same effect, although contract. Misrepresentation as a defense of the insurer to
the property is in the hands of the mortgagee, but any avoid liability is an affirmative defense and the duty to
act which, under the contract of insurance, is to be establish such defense by satisfactory and convincing
FACTS: A contract of group life insurance was executed performed by the mortgagor, may be performed by the evidence rests upon the insurer. In the case at bar,
between Great Pacific Life Assurance Corporation mortgagee therein named, with the same effect as if it Grepalife failed to clearly and satisfactorily establish its
(Grepalife; insurer) and Development Bank of the had been performed by the mortgagor. defense, and is therefore liable to pay the proceeds of
Philippines (DBP; creditor of the insured Dr. Wilfredo the insurance.
Leuterio). Grepalife agreed to insure the lives of eligible The insured Dr. Leuterio did not cede to the
housing loan mortgagors of DBP. mortgagee all his rights or interests in the insurance, the
policy stating that: In the event of the debtors death
On November 11, 1983, Dr. Wilfredo Leuterio, a before his indebtedness with the Creditor [DBP] shall As to Grepalife’s claims that there was no evidence
physician and a housing debtor of DBP applied for have been fully paid, an amount to pay the outstanding as to the amount of Dr. Leuterios outstanding
indebtedness to DBP at the time of the mortgagors travellers Insurance as the insurer of the truck, but the Any person having any claim upon the policy issued
death. The Supreme Court held: Grepalife’s claim is latter failed to act on petitioner's claim. pursuant to this chapter shall, without any unnecessary
without merit. A life insurance policy is a valued policy. delay, present to the insurance company concerned a
Unless the interest of a person insured is susceptible of Then petitioner filed a complaint against the private written notice of claim setting forth the amount of his
exact pecuniary measurement, the measure of indemnity respondent, the driver and the owner of the truck. The loss, and/or the nature, extent and duration of the injuries
under a policy of insurance upon life or health is the sum RTC rendered a decision in favor of the petitioner and sustained as certified by a duly licensed physician.
fixed in the policy. The mortgagor paid the premium ordered private respondent to pay petitioner the amount Notice of claim must be filed within six months from date
according to the coverage of his insurance, which states paid to PTCI, but dismissed the complaint as against the of the accident, otherwise, the claim shall be deemed
that: The policy states that upon receipt of due proof of other two defendants. waived. Action or suit for recovery of damage due to loss
the Debtors death during the terms of this insurance, a or injury must be brought in proper cases, with the
death benefit in the amount of P86,200.00 shall be Commission or the Courts within one year from date of
paid.In the event of the debtors death before his On appeal, the CA affirmed the finding of the RTC that it accident, otherwise the claimant's right of action shall
indebtedness with the creditor shall have been fully paid, was the negligence and recklessness of Alfredo Sion, prescribe. [Emphasis supplied].
an amount to pay the outstanding indebtedness shall first the driver of the Isuzu Cargo Truck, which led to the
be paid to the Creditor and the balance of the Sum vehicular accident. The CA also held that as the insurer
of the truck, private respondent is liable to herein In the case of Summit Guaranty & Insurance Co., Inc. v.
Assured, if there is any shall then be paid to the De Guzman [G.R. Nos. 50997, L-48679, L-48758, June
beneficiary/ies designated by the debtor. Furthermore, petitioner as the subrogee to all the rights and causes of
action of the owner of the damaged Toyota Land Cruiser. 30,1987,151 SCRA 389.] which involves similar facts, in
DBP foreclosed in 1995 their residential lot, in rejecting the insurance company's defense of
satisfaction of mortgagors outstanding loan. Considering Nevertheless, the CA dismissed the complaint on the
ground that petitioner's cause of action had prescribed prescription, the Court held that:
this supervening event, the insurance proceeds shall
inure to the benefit of the heirs of the deceased person because the complaint was not filed until almost
or his beneficiaries. Equity dictates that DBP should not seventeen (17) months after the accident. Petitioner company is trying to use Section 384 of the
unjustly enrich itself at the expense of another (Nemo Insurance Code as a cloak to hide itself from its
cum alterius detrimenio protest). Hence, it cannot collect Petitioner company contends that the finding of liabilities. The facts of these cases * evidently reflect the
the insurance proceeds, after it already foreclosed on the respondent court that its cause of action had prescribed deliberate efforts of petitioner company to prevent the
mortgage. is erroneous since the one-year prescriptive period under filing of a formal action against it. Bearing in mind that if it
Section 384 of the Insurance Code is counted not from succeeds in doing so until one year lapses from the date
the date of the accident but from the date of the rejection of the accident it could set up the defense of prescription,
of the claim by the insurer. Petitioner further argues that petitioner company made private respondents believe
27. BANKERS INSURANCE CORP. (Formerly Country that their claims would be settled in order that the latter
Bankers Insurance & Surety Co. Inc.) vs. THE even assuming that the one-year prescriptive period
should be counted from the date of the accident, the will not find it necessary to immediately bring suit. In
TRAVELLERS INSURANCE AND SURETY CORP., and violation of its duties to adopt and implement reasonable
THE HONORABLE COURT OF APPEALS, G.R No. running of the period of prescription was interrupted
when petitioner filed a notice of claim with respondent standards for the prompt investigation of claims, and with
82509 August 16, 1989 manifest bad faith, petitioner company devised means
Jean insurance company since under the Civil Code an extra-
judicial demand is sufficient to interrupt the running of the and ways of stalling settlement proceedings.
prescriptive period.
FACTS: To prevent the insurance company from evading its
ISSUE: responsibility to the insured through this clever scheme,
A vehicular accident occurred involving a Toyota Land and to protect the insuring public against similar acts by
Cruiser owned by Philippine Technical Consultants Inc. Whether or not petitioner's cause of action had
prescribed. --- NO! other insurance companies, the Court held that the one-
(PTCI) and an Isuzu Cargo Truck registered in the name year period under Section 384 should be counted not
of Avelino Matundan. The Toyota Land Cruiser, which from the date of the accident but from the date of the
was driven by Norlito R. Limen had stopped at a red light HELD: rejection of the claim by the insurer. The Court further
along Epifanio de los Santos Avenue when it was There is no dispute that respondent insurance held that it is only from the rejection of the claim by
bumped from behind by the Isuzu Cargo Truck driven by company is liable as the insurer of the Isuzu Cargo Truck the insurer that the insured's cause of action
Alfredo Sion. The Toyota Land Cruiser suffered and should reimburse to petitioner the amount paid by accrued since a cause of action does not accrue
extensive damage so that its owner declared a total loss the latter to PTCI for the damage sustained by the until the party obligated refuse, expressly or
and claimed the proceeds of the insurance policy issued Toyota Land Cruiser. impliedly, to comply with its duty.
by petitioner Country Bankers Insurance Corporation.
Finding the claim to be meritorious, petitioner paid PTCI Whether or not petitioner's cause of action had
the amount of eighty-three thousand four hundred In the instant case, petitioner sent a notice of claim to
prescribed. respondent insurance company as early as July 26, 1979
seventy pesos (P83,470.00).lâwphî1.ñèt As subrogee to
all rights and causes of action of PTCI, petitioner or two months after the accident. This was followed by a
demanded reimbursement from the driver and owner of Section 384 of the Insurance Code (prior to its letter dated August 3, 1979 urging respondent insurance
the Isuzu Cargo truck and from private respondent amendment by B.P. 874) provides that: company to take it appropriate action" on petitioner's
claim. However, it was only a year later, on August 3,
1980 that respondent replied to petitioner's letter
informing it that they could not take appropriate action on
petitioners claim because the attending adjuster was still Any person having any claim upon the policy issued
negotiating the case. Two months later, when pursuant to this chapter shall, without any unnecessary
respondent insurance company still failed to act on its delay, present to the insurance company concerned a
claim, petitioner filed the present case in court. During written notice of claim setting forth the nature, extent and
the hearing before the RTC, respondent insurance duration of the injuries sustained as certified by a duly
company never raised the defense of prescription. It was licensed physician. Notice of claim must be filed within
only on appeal that Section 384 of the Insurance Code six months from date of the accident otherwise, the claim
was invoked by respondent insurance company and the shall be deemed waived. Action or suit for recovery of
CA, relying on the plain language of the law, dismissed damage due to loss or injury must be brought in proper
the case on the ground of prescription. cases, with the Commissioner or the Courts within one
year from denial of the claim, otherwise the claimant's
In the light of the Court's decision in the Summit right of action shall prescribe. [Emphasis supplied].
case, respondent insurance company can no longer
invoke Section 384 to defeat petitioner's claim. As
aforestated, it was precisely to prevent unscrupulous
insurance companies from using Section 384 in
evading their responsibilities that the Court applied
Section 384 strictly against insurance companies in
the Summit case.

The requirement that any claim or action for recovery of


damage under an insurance policy must be brought
within one year from the date of the accident was
intended to ensure that suits be brought by the insured
while evidence as to the origin and cause of destruction
have not yet disappeared.lâwphî1.ñèt This is to enable
the insurance companies to make proper assessment of
whether or not the insured can recover and, if so, to
determine the amount recoverable. However, where, as
in this case, the delay in bringing the suit against the
insurance company was not caused by the insured
or its subrogee but by the insurance company itself,
it is unfair to penalize the insured or its subrogee by
dismissing its action against the insurance company
on the ground of prescription. The latter should bear
the consequences of its failure to act promptly on
the insured's claim. Under the law, insurance
companies are duty bound to adopt and implement
reasonable standards for the prompt, fair and
equitable settlement of claims [Section 241,
Insurance Code].

Therefore, considering the attendant facts of this case,


the Court finds that the doctrine laid down in the Summit
case is applicable, and accordingly holds that petitioner's
cause of action has not prescribed.

It might not be amiss to state that Section 384 was


amended in 1985 by Batas Pambansa Blg. 874. The
amendment was inserted by the then Batasang
Pambansa after realizing that Section 384 of the
Insurance Code has created so many problems for the
insuring public [Summit, supra at p. 398]. Thus, as
amended, the law now provides that:

Das könnte Ihnen auch gefallen